Решение задач по фотографии онлайн: Кампус – решение примеров и задач по фото онлайн

Решение задач по математике по фото онлайн

Не каждому ребенку успешно даются примеры и задачи по математике. Для них в сети есть программы и приложения, решающие примеры и задачи по фотографии онлайн. Их достаточно много, поэтому будут рассмотрены только лучшие. Из статьи вы узнаете, как ими пользоваться.

Содержание

  1. Как решить задачу по математике по фото в Photomath
  2. Решение задач в приложении Google Lens
  3. Поиск ответа на задачи в Mathway
  4. Mathway достепен онлайн
  5. Математический сканер по фото
  6. Видеоинструкция

Как решить задачу по математике по фото в Photomath

Новая тема по школьным предметам далеко не каждым учеником сразу воспринимается. Ребёнку нужно решить несколько задач с примерами, объяснить принцип функций и тогда через какое-то время материал будет усвоен. Таким виртуальным репетитором может быть мобильное приложение Photomath для Android и Айфона. Программа не только решит задачу по картинке, но и подскажет все действия решения. Поможет понять пример, разложив его на действия.

  1. Откройте учебник или любой другой источник примера;
  2. Запустите приложение и откройте в нём камеру;
  3. Наведите объектив камеры на задачу и сделайте снимок. Нажмите для этого на красную кнопку внизу экрана;
  4. Затем нажмите на кнопку «Показать шаги решения». И на экране появятся действия, с помощью которых программа решала данную задачу.

Как видите, всё легко и просто работает. Любая самая сложная задача может быть разложена на простые шаги для решения. В приложении Photomath поддерживается более тридцати языков. Доступны разнообразные интерактивные графики. Камера приложения легко распознаются и текстовый текст из учебника, и рукописные записи конспекта. Кроме этого, в приложении доступно несколько методов решения одной и той же задачи.

Это интересно: решение уравнений по фото онлайн.

Решение задач в приложении Google Lens

Математические примеры научилось решать популярное приложение — Google Lens. До этого программа легко справлялась с поиском любых предметов в Интернете. Способно переводить на лету текст с одного языка на другой.

Выбор текста в Гугл Ленс

Разработчики из компании Гугл решили помочь студентам и школьникам и реализовали в своей программе настоящий онлайн-решебник. Теперь камера способна щёлкать математически примера в одну секунду. Запустив камеру Гугл, наведите её на задачу и сделайте фото.

Копия примера будет найдена в Интернете вместе с решением. Для того, чтобы пример был лучше рассмотрен объективом, сместите фиксирующее окно в место, где расположено условие в учебнике или конспекте.

Мобильное приложение Google Lens способно определять и другие предметы. Вы с лёгкостью сможете найти в сети по картинке любой костюм, платье. Узнать больше о достопримечательностях города, в который приехали на экскурсию. Перевести любые указатели и таблички с иностранного языка.

Google Lens умеет сканировать штрих и QR коды. По картинке определяет название блюд в кафе и ресторанах. Всё что ему нужно для полноценной работы — это подключение к сети Интернет. Приложение не сохраняет изображения, по которым вы ищите информацию. После загрузки и сканирования они удаляются с сервиса без индексирования. Подробнее об этом можно прочитать в пользовательском соглашении.

Читайте также: как написать корень на клавиатуре.

Поиск ответа на задачи в Mathway

Школьники давно мечтали о таких программах, которые бы делали домашнее задание за них. Наконец-то они появились. И способны сканировать любые примеры при помощи камеры на мобильном телефоне. Нужно лишь навести объектив смартфона на сложный пример и нажать кнопку для создания фото.

Сканирование примера в Mathway

В приложении Mathway всё происходит в автоматическом режиме. Скачать программу можно для телефонов с Android и Айфона.

Программа Mathway легко решает математические задачи при помощи фотографии онлайн. Доступ в Интернет необходим, чтобы оно проверило свою базу и попыталось найти ответ. Офлайн доступны другие функции, но решение по изображению невозможно. Использование приложения бесплатно для всех пользователей. В нём также масса других возможностей.

  • Решение примеров математики для начальных классов;
  • Справляется с задачами по тригонометрии и алгебре;
  • Выводит всю статистику решения и дополнительную информацию;
  • Решает задачи по химии и линейной алгебре;
  • Способно создавать графики любой сложности.

Приложение было высоко оценено различными западными СМИ и популярными Интернет-компаниями, в числе которых: Yahoo!, CNET, Lifehack и другие. Главными её качествами отмечают: простоту в использовании, эффективность в решении задач, быстрый поиск решения. Программа не просто выводит результат, но и помогает понять, какие действия для её решения предпринимались.

Mathway достепен онлайн

Кроме мобильного приложения Mathway есть ещё веб-сайт, на котором легко можно решать задачи по математике, сидя дома за компьютером. Попробуйте этот инструмент, если с другими возникли проблемы. Ресурс находится по адресу: Mathway.com.

Веб-интерфейс Mathway

При переходе пользователь видит окно с приглашением к вводу данных. Программа уже готова решать примеры по фотографии или при вводе информации в текстовой форме.

Написание условия задачи

Для учеников, которые впервые на сайте, предлагается текстовая инструкция по работе с сайтом. Найти её можно в нижней части, в левом углу.

Выбор школьной дисциплины

Стоит убедиться, что в данный момент выбрать нужный предмет. Нажмите на кнопку меню вверху и выберите другой, если это необходимо.

  1. Чтобы сфотографировать пример, нажмите на иконку фотоаппарата с плюсом;
  2. Затем наведите веб-камеру на пример или поднесите книгу к камере ноутбука. На экране будет виден результат;
  3. На виртуальной клавиатуре можно ввести текстовые данные для решения задачи;
  4. Данные также можно набирать на обычной клавиатуре, а математически знаки добавлять из виртуальной;
  5. Программе можно предоставить скан страницы с задачей, если камера на компьютере отсутствует.

    Список предметов

Ресурс будет полезен всем учащимся, студентам и людям разных профессий. Помощь по сайту можно получить в разделе справки, которая находится в меню сайта. Для пользователей, которые не желают забивать память мобильного устройства разными приложениями, можно пользоваться онлайн сайтом Mathway даже через мобильный браузер. Все функции, которые доступны в приложении, можно применять в веб-версии.

Математический сканер по фото

Найден ещё один полезный инструмент для решения математических задач — Математический сканер. Эта программа умеет работать без подключения к сети Интернет. И выдавать решения не хуже остальных подобных программ из App Store или Google Play. Основным её инструментом является камера. Использовать в этом приложении лучше только печатный текст из учебников. Чтобы программа смогла определить сложность задачи и показать её решения. Иногда не справляется с написанным текстом от руки.

  • Программа «Математический сканер» также способна отображать полное решения задач со схемой действий;
  • Есть режим, в котором программа показывает каждый шаг детально, который она принимала для получения результата задачи;
  • Решает любые примеры на вычитание, проценты, сложение, умножение и прочее;
  • В ней есть возможность создавать графики и таблицы.

В программе встроен обычный калькулятор с дополнительными кнопками, который может быть использован из главного меню. А также имеет множество различных конвертеров. Для использования фото-сканера выберите его в приложении и сфотографируйте пример. Убедитесь в том, что все части его условия поместились в объектив. После чего можно увидеть результат решения математической задачи.

Видеоинструкция

Попробуйте использовать одно из описанных приложений, чтобы решить задачу по математике по картинке. Если возникли трудности с примерами на проценты, посмотрите это видео.

Решение примеров и задач по фото онлайн

Автор Денис Озерянский На чтение 7 мин Опубликовано Обновлено

Развитие информационных технологий здорово упростило жизнь во многих аспектах. В том числе и процесс образования. Сегодня в интернете можно найти ответ на любые вопросы из школьных и университетских учебников. Но до недавнего времени единственной сложностью оставались задачи фундаментальных предметов.

Сейчас, имея под рукой смартфон, можно найти решение по фото или картинке почти любого примера. Конечно, если установить правильно приложение. А те, что не поддаются машинной логике, доверить более опытным людям из интернета. А если вы еще не знаете, как решить пример по фото, но хотите научиться, то эта статья точно для вас.

Содержание

  1. Приложения для смартфона
  2. Maple Калькулятор
  3. Microsoft Math Solver
  4. Math Scanner
  5. Photomath
  6. Сервис Mathway для решения примеров по фото онлайн
  7. Платные сервисы для решения задач
  8. HomeWork
  9. Все сдал!

Приложения для смартфона

Если открыть Play Market по соответствующему запросу, можно увидеть внушительный список приложений. К сожалению большинство из них не всегда корректно распознают пример на фото. Трудно сказать в чем проблема: то ли сканер плохо распознает пример, то ли почерк непонятный. Поэтому пришлось повозится, чтобы отобрать только качественные приложения, которые отлично справляются со всеми этапами работы.

Maple Калькулятор

Maple Калькулятор – отличный инструмент для работы с математическими примерами. Приложение решает задачи не только школьного уровня, но также и примеры университетского уровня. Оно русифицировано и полностью бесплатно и, как ни странно, в нем полностью отсутствует реклама. Такое еще поискать надо!

Чтобы найти решение примера по фото с помощью Maple Калькулятора, выполните следующее:

  1. Запустите приложение и тапните по иконке камеры на главном экране (при первом запуске программа запросит доступ к фотокамере, нажмите «Подтвердить»).
  2. Наведите окно сканера на пример, таким образом, чтобы он весь попал в окно и сделайте фото.
  3. Дождитесь пока приложение отсканирует пример, при необходимости отредактируйте выражение, нажав на значок карандаша рядом с записью.

Если решаете пример из геометрии, можно посмотреть график функции. Для этого разверните пункт «Двухмерный график». Только он почему-то строится трехмерный, но общий смысл понятен.

Microsoft Math Solver

Приложение Math Solver не просто выдает готовый результат вычислений, но также показывает шаги решений и итоговый график. Что будет весьма полезно для решения домашних работ по математике и алгебре. А дополнительные функция подробного описания хода решения будет особенно полезна для решения задач таких предметов как физика и геометрия.

Чтобы решить пример по фото выполните следующие действия:

  1. Запустите приложение (при первом запуске выберите язык и пролистайте ознакомление с основными функциями, при желании ознакомьтесь с ними).
  2. Сфотографируйте пример.
  3. После того как приложение выполнит сканирование, потяните появившееся окошко вверх, чтобы увидеть подробности решения и правильный ответ.
  4. Если хотите посмотреть шаги решения, нажмите соответствующую кнопку на экране.

Math Solver отлично русифицировано, бесплатно и не содержит рекламы. От предыдущего отличается только тем, что не умеет строить трехмерные графики.

Math Scanner

Math Scanner бесплатное нерусифицированное приложение со встроенной рекламой. Но простота интерфейса и ненавязчивость рекламы никак не мешают пользоваться им при слабом знании английского. К тому же у него высокая точность сканирования.

Чтобы решить пример с помощью Math Scanner следуйте инструкции ниже:

  1. Запустите приложение и пропустите рекламу, чтобы открыть основное меню.
  2. Наведите сканер на пример, так, чтобы он целиком попал в окно и нажмите кнопку активации.
  3. Дождитесь пока приложение выполнит сканирование и подтвердите полученный результат (если Math Scanner понял уравнение неверно, перепишите пример аккуратнее и повторно запустите сканирование).

В следующем окне вы увидите правильный ответ и этапы решения.

В отличии от описанных выше приложений, Math Scanner не строит графики. Зато с его помощью вы всегда сможете посмотреть похожие задачи.

Photomath

Photomath – это одно из двух самых известных приложений для решения примеров по фото. Это не просто удобное приложение, облегчающее подготовку домашнего задания для школьников и студентов, а полноценный обучающий портал с разделами для учеников, родителей и учителей.

Чтобы решить пример или уравнение с помощью Фотомач сделайте следующие действия:

  1. При первом запуске укажите свой возраст и категорию (учитель, ученик, родитель). Это нужно, чтобы приложение правильно показывало рекомендации и по необходимости подбирало обучающий материал.Больше всего полезной информации Photomath показывает в категории «Ученик».
  2. Отсканируйте записанный пример и дождитесь пока приложение решит его. Если хотите посмотреть шаги по решению, то нажмите соответствующую кнопку в окне программы.

Среди окон с результатами вы найдете не только шаги решения, но также график функции, разные способы записи уравнения и пояснения к каждому этапу вычислений. А для сложных примеров длинные и укороченные способы расчета.

Сервис Mathway для решения примеров по фото онлайн

Без сомнений, пользоваться смартфоном для решения примеров удобно. Но что делать, если доступа к гаджету нет, а пример решить нужно? На помощь придет онлайн сервис Mathway. А сфотографировать пример можно, например, на веб камеру. Главное, чтобы изображение было читаемым.

Принцип работы сервиса аналогичен приложениям для смартфона. Чтобы решить пример по фото выполните следующие действия:

  1. Сфотографируйте пример и загрузите его на компьютер.
  2. На сайте сервиса выберите необходимый раздел науки, которой соответствует пример (необязательное действие, если вам не нужны подробности вычислений).
  3. Через форму загрузите пример на сайт.
  4. Если на фотографии записано несколько примеров или присутствует текст, выделите в специальном окне нужную задачу. А затем нажмите кнопку «Отправить» (иконка в форме бумажного самолетика).
  5. Если сервис верно распознал выражение, еще раз нажмите кнопку «Отправить» в правом нижнем углу экрана.
  6. Дождитесь пока сервис выполнит расчет. Если нужны подробности решения, кликните по надписи: «Нажмите для просмотра шагов…».

Платные сервисы для решения задач

Как решать примеры по фото понятно. Описанных выше сервисов более чем достаточно, чтобы справиться с задачами по алгебре, физике и даже химии за 5-7 класс. Однако домашние задания для старших классов и тем более университетские по высшей математике могут быть настолько сложными, что даже правильно записать равенство не всегда получится. Конечно можно открыть решебник или повторить пройденный материал. И, в принципе, это правильное решение. Но не всегда найдется время.

В таком случае автоматизированные сервисы не помогут, а значит нужно обращаться к преподавателям по соответствующему предмету. И если среди знакомых таковых нет, можно воспользоваться услугами сервисов, где задачи решают люди.

HomeWork

Сервис HomeWork устроен по принципу биржи. После регистрации вы оставляете задание с фотографией задачи из учебника и коротким описанием того, что нужно сделать. Далее сотрудник сервиса проведет предварительную оценку и передаст задачу на решение специалистам. Если цена вас устроит последний приступит к решению. Процесс не самый быстрый, в зависимости от сложности задачи может потребоваться от получаса до 3-4 часов и даже больше.

Чтобы воспользоваться услугами сервиса следуйте инструкции ниже:

  1. На главной странице сайта выберите тип работы и укажите адрес вашей электронной почты, а затем нажмите «Продолжить».
  2. В следующем окне укажите предмет, кратко опишите что нужно сделать и загрузите фото задачи или примера, перетащив фото в специальное поле (цифра 3 на скриншоте ниже).
  3. Самый неприятный момент. Для завершения регистрации необходимо указать свой номер телефона, куда будет направлена цена решения (спамить будут гарантированно, поэтому лучше указывать дополнительный номер, если таковой имеется).

Это же действие завершит создание аккаунта на сервисе. Спустя 10-15 минут менеджер пришлет смс с указанием стоимости решения. Она же появится в личном кабинете. Если цена вас устраивает, подтвердите выполнение работы и оплатите услуги.

Все сдал!

Принцип работы сервиса Все сдал! такой же, как и у предыдущего, только здесь не просят номер телефона. Что избавляет от головной боли со спам-звонками.

Чтобы оставить заявку на решение задачи, сделайте следующее:

  1. Выполните регистрацию на сайте (укажите e-mail и установите пароль), либо воспользуйтесь упрощенной формой для входа.
  2. В личном кабинете создайте задание.
  3. Заполните поля формы, загрузите фото задачи или примера, а затем нажмите кнопку «Разместить заказ».
  4. Дождитесь пока сервис пришлет уведомление о выполнении работы, который появится в разделе «Мои заказы».

После оплаты вы сможете увидеть решение и при необходимости скачать его.

Обложка: кадр из промо видео Maple Calculator

Проблемы с фотографией? Получите исправления здесь!

Если вы увлекаетесь фотографией, возможно, вы уже знаете, насколько сложно сделать один идеальный снимок. Иногда, даже после того, как мы все сделали правильно, мы сталкиваемся со многими нежелательными проблемами, такими как красные глаза, размытие, передержка и другие проблемы.

Хорошей новостью является то, что вы можете решить распространенные проблемы с фотографией с помощью многочисленных инструментов. Здесь я познакомлю вас с этими распространенными проблемами с фотографией, а также предоставлю несколько надежных решений для их устранения.

В этой статье

01 Часть 1: 10 самых распространенных проблем с фотографией

02 Часть 2: Как решить распространенные проблемы с фотографией?

Часть 1: 10 наиболее распространенных проблем с фотографией

Прежде чем устранять распространенные проблемы с фотографией, важно знать их основы и то, что могло вызвать эти проблемы.

Проблема 1: размытые фотографии или размытые пятна

Получение размытых фотографий или объектов — одна из наиболее часто встречающихся проблем в фотографии. В некоторых случаях вся фотография размыта, а иногда размыты определенные участки.

Эта проблема может быть вызвана тем, что объектив не очищается из-за скопления грязи, пыли или запотевания. Кроме того, если вы пытаетесь запечатлеть движущийся объект или ваша камера дрожит, это также может сделать ваши фотографии размытыми. Чтобы избежать этой проблемы, вы можете отрегулировать уровень ISO вашей камеры, щелкнув движущийся объект.

Проблема 2: Плохая или неправильная цветопередача

Иногда при съемке в помещении цвета на фотографиях могут отличаться. Например, фиолетовый объект может выглядеть на ваших фотографиях темно-бордовым. Хотя для этого нет единой фиксированной причины, это может быть вызвано неправильными настройками цветового баланса в вашей камере, поскольку явление цветокоррекции является распространенным явлением.

Таким образом, чтобы исправить распространенные проблемы с фотографией, связанные с плохой цветопередачей, вы можете просто настроить параметры цветового баланса. Кроме того, вы также можете включать и выключать вспышку, чтобы объекты на фотографиях выглядели более естественно.

Проблема 3: у людей на фотографиях красные глаза

Это старая проблема, которая до сих пор беспокоит многих фотографов. В идеале, если вы фотографируете в помещении и используете вспышку, это может вызвать эффект красных глаз (когда вспышка отражается при щелчке по фотографиям).

Чтобы решить эту проблему, вы можете попробовать отключить вспышку или отрегулировать общее освещение в комнате. Вы также можете попросить людей не смотреть прямо в объектив при включенной вспышке или даже использовать инструмент редактирования, чтобы избавиться от красных глаз.

Проблема 4: зернистые, туманные или зашумленные изображения

Еще одна распространенная проблема, с которой вы можете столкнуться во время фотосъемки, — наличие нежелательных зерен или туманных областей. Проблема обычно возникает, когда мы щелкаем наши фотографии в режиме высокой чувствительности, а камера не может обработать изображение.

Кроме того, если у вас появляются шумы на фотографиях, это также может быть связано с изменением настроек вашей камеры или ее плохим качеством. Вы можете попробовать настроить параметры ISO или чувствительности вашей камеры для разрешения зернистых изображений.

Если вы хотите узнать больше о том, почему фотографии зернистые, этот веб-сайт может дать вам исчерпывающий ответ.

Проблема 5: Неправильный баланс белого

Возможно, вы уже знаете, что баланс белого — один из важнейших параметров изображения. Хотя, если вы фотографируете в ручном режиме, может возникнуть проблема с балансом белого.

После этого ваши фотографии могут выглядеть слишком белыми или слишком темными. Самый простой способ исправить это — посетить настройки баланса белого в вашей камере и отрегулировать их вручную.

Проблема 6: искаженные или вытянутые фотографии

Во время съемки или фотосъемки вы можете столкнуться со всевозможными ошибками искажения. Например, после обработки изображения некоторые объекты могут казаться вытянутыми или составными.

Во время фотосъемки старайтесь держать камеру неподвижно или даже поставить ее на штатив. Кроме того, вы можете попытаться не слишком сильно уменьшать сцену, так как это может привести к нежелательным искажениям на ваших фотографиях.

Проблема 7: Передержка или недодержка фотографий

Если вы щелкаете фотографии в ручном режиме, вы можете столкнуться с проблемами, связанными с их экспозицией. Например, если ваши фотографии недоэкспонированы, они могут казаться слишком темными, а переэкспонированные фотографии — слишком яркими.

Самый простой способ решить эту проблему — настроить параметры экспозиции камеры. Вы можете просто увеличить или уменьшить уровень экспозиции в соответствии с вашими требованиями, чтобы получить правильную насыщенность изображения.

Проблема 8: линия горизонта не прямая

Это особая проблема, возникающая при неправильном размещении камеры. В идеале, если ваша камера установлена ​​на штатив и наклонена, то горизонт на снимке также не будет выглядеть прямым.

К счастью, вы можете решить эти распространенные проблемы с фотографией, правильно расположив камеру. В большинстве камер нового поколения вы также можете получить доступ к встроенному выравнивателю или стабилизатору, чтобы решить эту проблему.

Проблема 9: Нечеткие или шумные фотографии в темноте

Многие фотографы-любители сталкиваются с проблемами, из-за которых получаются нечеткие пятна или шумные щелчки в темноте. Во-первых, вы можете попробовать включить или выключить вспышку в вашей камере, а также можете попробовать щелкнуть изображения в ночном режиме

Кроме того, вы можете проверить настройки экспозиции вашей камеры и вручную установить фокус на определенных объектах. Вы также можете обнаружить источник света в сцене и делать снимки в том же направлении (а не в противоположном). Таким образом, вы можете устранить любые нечеткие пятна во время фотосъемки в темноте.

Проблема 10. Серые области или пятна на фотографиях

Наконец, вы можете получить серые области на фотографиях при щелчке, редактировании или передаче. В идеале может быть несколько причин, связанных с настройками вашей камеры или проблемами редактирования, которые могут вызвать это.

Чтобы решить эти распространенные проблемы с фотографией, вам необходимо заранее убедиться, что ваша камера достаточно заряжена. Кроме того, вам следует избегать тряски камеры при щелчке фотографий. Также рекомендуется не изменять формат ваших фотографий вручную, чтобы избежать этой проблемы.

Часть 2. Как решить распространенные проблемы с фотографией?

Как видите, вы можете столкнуться с множеством различных проблем с фотографией при нажатии или обработке изображений. Однако, если вы хотите исправить распространенные проблемы с фотографией, вы можете рассмотреть следующие предложения.

Автоматическое решение: используйте Wondershare Repairit для устранения всех видов проблем с поврежденными фотографиями

Если вы ищете автоматическое решение для устранения распространенных проблем с фотографиями, вы можете использовать специальный инструмент, такой как Wondershare Repairit. Приложение может исправить большинство перечисленных выше проблем с несколькими другими проблемами, связанными с вашими изображениями. Если ваши фотографии не загружаются, имеют пиксельные области, серые пятна или иным образом повреждены, тогда Repairit будет идеальным решением.

Чтобы устранить распространенные проблемы с фотографиями с помощью Repairit, можно предпринять следующие шаги:

Попробуйте бесплатно

Попробуйте бесплатно

Шаг 1: Загрузите любую поврежденную фотографию на Repairit

Во-первых , вы можете просто запустить Wondershare Repairit в своей системе и перейти к функции восстановления фотографий сбоку. Здесь вы можете нажать кнопку «Добавить», чтобы вручную загрузить поврежденные фотографии.

После загрузки поврежденных фотографий вы можете просто проверить их предварительный просмотр и начать процесс восстановления.

Шаг 2. Восстановите и просмотрите фотографии

Приложение автоматически выполнит быстрое восстановление ваших фотографий и уведомит вас об этом.

Теперь вы можете нажать кнопку предварительного просмотра или значок глаза, чтобы получить мгновенный предварительный просмотр ваших фотографий и проверить результаты восстановления.

Если ваши фотографии сильно повреждены, приложение сообщит вам об этом и отобразит следующую подсказку.

Теперь вы можете выполнить расширенное восстановление ваших фотографий (загрузив образец изображения, который должен быть сделан на том же устройстве и иметь тот же формат, что и поврежденная фотография). Хотя расширенный ремонт Repairit займет больше времени, его результаты также будут превосходными.

Шаг 3. Экспортируйте восстановленные фотографии

Наконец, вы можете предварительно просмотреть восстановленные фотографии и просто нажать кнопку «Сохранить» (или кнопку «Сохранить все»). Теперь вы можете выбрать место, куда ваши фотографии будут экспортированы в вашей системе с помощью Repairit.

Следуя этому простому подходу, вы можете одновременно восстанавливать несколько фотографий с помощью Wondershare Repairit.

Ручные решения для устранения проблем с фотографиями

Кроме того, если вы хотите внести всевозможные изменения в свои фотографии вручную, вы можете использовать редактор изображений, такой как Photoshop или Lightroom.

Adobe Photoshop

Photoshop — наиболее широко используемый редактор растровых изображений от Adobe, обладающий множеством функций. Используя Photoshop, вы можете вылечить любую поврежденную область на своих фотографиях или настроить общую цветокоррекцию изображения. Вы также можете вручную настроить экспозицию или баланс белого ваших изображений и выполнить множество других правок.

Adobe Lightroom

Lightroom — это комплексное программное обеспечение для организации и редактирования изображений, которое можно использовать для внесения незначительных изменений. Вы можете вручную масштабировать изображения и настраивать их важные параметры, такие как насыщенность, яркость, температура, баланс, цветокоррекция и так далее. Есть также множество вариантов для лечения изображения или применения легкодоступных фильтров.

Пора заканчивать

Я уверен, что после прочтения этого поста вы сможете самостоятельно решить распространенные проблемы с фотографией. От зернистых изображений до размытых объектов и серых областей до передержки — с вашими фотографиями может быть так много проблем. Тем не менее, с помощью такого инструмента, как Wondershare Repairit , вы можете мгновенно исправить все эти распространенные проблемы с поврежденными фотографиями, выполнив процесс клика. Кроме того, если вы хотите получить индивидуальные результаты, вы всегда можете использовать любой надежный редактор изображений.

Бесплатная пробная версия

Бесплатная пробная версия

Фотография как процесс решения проблем — УЛИЧНЫЕ СИЛУЭТЫ

Прежде чем я отважусь на фотопрогулку, я должен сначала выбрать способ съемки. Будет ли это пленка или цифровой захват? То, как это будет решено, зависит от потребностей моего рабочего процесса, то есть от того, могу ли я ждать, пока пленка будет проявлена, или нет. На практике я снимаю на цифру для работы, потому что уверенность, обеспечиваемая цифровым изображением в момент захвата, является более точной и безошибочной. Кроме того, удобство немедленного удовлетворения позволяет мне работать с файлами изображений без каких-либо задержек. Тем не менее, я обычно снимаю на пленку для личного удовольствия.

Если выбрана пленка, я выберу цветную или черно-белую? Если выбран цвет, я выберу положительный или отрицательный? То, что я выбираю, зависит от внешних факторов, не зависящих от меня, таких как условия освещения для времени суток и погоды, ограничения, обнаруженные в моем местоположении, и общая степень случайности цветов окружающей среды. Другими словами, если цвета будут тусклыми или конфликтующими, я буду снимать на черно-белую пленку. Если цвета будут яркими и дополняющими друг друга, я буду снимать слайды. Для всего остального буду снимать негативы.

После того, как мы разобрались со средством захвата, следующий порядок действий — распознать поставленную задачу. По сути, это выяснение того, что и как фотографировать. Многие энтузиасты обычно подходят к этому шагу с неуверенностью из-за отсутствия четкой цели и методологии. Таким образом, большинство рекреационных фотографий имеют тенденцию выглядеть неуклюже в представлении. В основном это происходит потому, что многие не понимают, что фотография — это процесс решения проблем, поскольку большинство не знает, какова цель фотографии.

Kodak Tri-X 400 с фокусным расстоянием 28 мм — взаимодействие с окружающей средой благодаря опоре на бамбуковые леса. Композиционные переменные включают строительные леса и фоновые вывески.

Kodak Portra 400 с фокусным расстоянием 35 мм. Взаимодействие с окружающей средой благодаря балансированию на краю закрытой витрины магазина. Композиционные переменные включают яркие цвета, граффити и горизонтальные линии.

Kodak Tri-X 400 с фокусным расстоянием 28 мм — взаимодействует с окружающей средой, прислонившись к дверному проему. Композиционные переменные включают прямолинейные линии дверных проемов, зданий и уклон дороги.

Kodak Portra 400 с фокусным расстоянием 35 мм — взаимодействие с окружающей средой, сидя на пожарном гидранте. Композиционные переменные включают кирпичную кладку, гармошку на витрине и решетки на металлической двери.

Kodak Tri-X 400 с фокусным расстоянием 28 мм — взаимодействие с окружающей средой путем слежения за почтовым ящиком. Композиционные переменные включают облупившуюся краску и почтовый ящик.

Kodak Tri-X 400 с фокусным расстоянием 28 мм. Взаимодействие с окружающей средой путем нажатия на дверную ручку витрины магазина. Захват автомобиля был ошибкой в ​​​​документации, поскольку он, похоже, конкурирует с предметом.

Цель фотографии — заполнить пустое пространство. Способ, которым это достигается, определяется композицией. Обычно это диктуется тем, как присутствие и расположение композиционных переменных (таких как формы, цвета и текстуры, присущие условиям фотосъемки) могут быть оптимизированы при захвате. Таким образом, цель хорошей композиции состоит в том, чтобы продуктивно заполнить пустое пространство композиционными переменными ради оптимизации предмета и контекста окружающей среды в презентации.

Композиционные переменные — это самая простая редукция компонентов, из которых состоит фотовозможность, поскольку они разбивают визуальное пространство в кадре. Из-за разницы контрастов в цветах, формах и текстурах, которые подвергаются воздействию доступного света, наличие линейных и нелинейных узоров можно использовать для устранения визуального напряжения пустых пространств — при условии, что практикуются надлежащие композиционные основы. Таким образом, на фотографии создается впечатление объема, что делает ее завершенной для глаз зрителя.

Другими словами, пустое пространство выглядит плоским, и слишком большое его количество в документации сделает захват изображения неполным из-за отсутствия объема. Из-за этого создается впечатление визуального напряжения для любой фотографии, которая заметно незавершена по композиции, учитывая распространенное мнение, что все хорошие фотографии завершены по композиции, а не заполнены пустым пространством. Таким образом, отсутствие объема — это отсутствие содержания. Таким образом, любая фотография, лишенная композиционных переменных, по сути неоптимизирована по замыслу.

Kodak Tri-X 400 с фокусным расстоянием 28 мм — ходьба в гору. Композиционные переменные включают текстуру стены, вентиляционные отверстия HVAC и окна.

Kodak Tri-X 400 с фокусным расстоянием 28 мм — взаимодействие с окружающей средой путем вытягивания надземного корня дерева. Композиционные переменные включают криволинейные надземные корни деревьев.

Kodak Tri-X 400 с фокусным расстоянием 28 мм — взаимодействует с окружающей средой, откидываясь на металлической скамье. Композиционные переменные включают фоновую роспись.

Kodak Tri-X 400 с фокусным расстоянием 28 мм. Композиционные переменные включают набор вытяжных блоков кондиционера.

Kodak Portra 400 с фокусным расстоянием 35 мм — взаимодействие с окружающей средой путем скольжения по перилам. Композиционные переменные включают лестницы с обесцвечиванием.

Kodak Portra 400 с фокусным расстоянием 35 мм — Поза дерева. Композиционные переменные включают беспорядок из фонового магазина.

Как только определена возможность фотографирования с достаточным количеством композиционных переменных, следующим моментом рассмотрения в хорошей композиции является расстояние съемки относительно выбранного фокусного расстояния объектива. Определение этого зависит от оптимизации описания предмета и контекста окружающей среды в документации. Другими словами, ваше расстояние съемки зависит от того, сколько объекта и фона вы хотите в кадре — ограничено фокусным расстоянием вашего объектива и условиями съемки возможности фотографирования.

Обычно в фотографии мы фотографируем людей. Конечно, есть и множество других жанров фотографии, а именно натюрморты, архитектурные и пейзажные. Тем не менее, именно в фотографии людей оптимизация кадрирования необходима как для объекта, так и для фона. Обычно для других жанров концентрация в кадрировании — это только объект или фон, точнее — объект — это фон (как в архитектурной и пейзажной фотографии) или фон гораздо менее важен (как в натюрморте).

При съемке людей на практике есть только два подхода. Либо вы зависите от работы с желающими участниками, либо вы зависите от возможностей с не желающими. Если ваша фотография зависит от невольных участников — как в случае с уличной фотографией — ваша композиция никогда не будет полностью такой, какой вы хотите ее представить. Незнающий объект никогда не будет правильно позировать или иметь желаемое выражение лица, расстояние съемки редко оптимизируется, или композиционные переменные редко будут полными.

Kodak Tri-X 400 с фокусным расстоянием 28 мм — взаимодействие с окружающей средой путем наступания на край дверного проема. Композиционные переменные включают текстуру стены, внешнюю сантехнику и пятна.

Kodak Tri-X 400 с фокусным расстоянием 28 мм — взаимодействие с окружающей средой путем поиска под отслаивающейся наклейкой с ценой. Композиционные переменные включают фоновые вывески.

Kodak Portra 400 с фокусным расстоянием 35 мм — композиционные переменные включают витрину слесаря… или это действительно слесарь…

Kodak Tri-X 400 с фокусным расстоянием 28 мм — взаимодействие с окружающей средой путем копирования жеста с вывески. Композиционные переменные включают ключи и фоновые обозначения.

Kodak Tri-X 400 с фокусным расстоянием 28 мм — взаимодействуйте с окружающей средой, играя с замком. Композиционные переменные включают в себя ключи и замки.

Kodak Portra 400 с фокусным расстоянием 35 мм. Композиционные переменные включают ярко-синий дверной проем.

Работая с желающими участниками, можно намного лучше контролировать композицию. Можно определить, какая часть объекта требуется в кадре. Это может быть все тело с головы до ног, или только до щиколоток, или только до бедер, бедер, талии, над ключицей или только лицо. А если только лицо, то с какого ракурса? Фронтальный, три четверти или боковой профиль? И будет ли голова поднята вверх или опущена вниз или будет поворачиваться в сторону под углом? Будет ли голова смотреть в том же направлении, что и тело?

А что тело будет делать в документации? Будет ли вес испытуемого приходиться на ведущую ногу или на обе ноги? Ноги будут вместе или широко расставлены? Будут ли обе руки лежать на высоком бедре, или они обе будут делать что-то еще, например, взаимодействовать с окружающей средой? Будет ли он держаться за что-то? Или субъект будет что-то делать? Сидишь? Склоняясь? Прыгать? Бег? Поза дерева? Или взаимодействие с окружающей средой? Открытие ваших вариантов поз — это преимущество сотрудничества.

Также возникает вопрос, сколько и какую часть кадра использовать при документировании предмета? Вы можете заполнить объект, используя весь кадр. Или вы можете привязать объект к верхней или нижней части и просто использовать часть кадра. Вы можете использовать три четверти, или две трети, или половину, или даже меньше, а остальная часть кадра захватывает фоновые композиционные переменные. В зависимости от ваших композиционных потребностей вы можете управлять отношением объекта к фону, регулируя масштаб объекта.

Kodak Portra 400 с фокусным расстоянием 35 мм — взаимодействие с окружающей средой благодаря висению на выступающем шесте.

Kodak Portra 400 с фокусным расстоянием 35 мм — взаимодействует с окружающей средой, сидя на другом пожарном гидранте. Композиционные переменные включают фоновую роспись и насыщенные цвета.

Kodak Tri-X 400 с фокусным расстоянием 28 мм

Kodak Portra 400 с фокусным расстоянием 35 мм — взаимодействует с окружающей средой, глядя в боковые зеркала грузовика. Композиционные переменные включают розовый грузовик и его содержимое.

Kodak Portra 400 с фокусным расстоянием 35 мм

Kodak Portra 400 с фокусным расстоянием 35 мм — взаимодействие с окружающей средой путем подвешивания на шесте.

Еще один вариант, который стоит рассмотреть, — это кадрирование более чем одного предмета. Если да, то сколько предметов? Будут ли они все вместе взаимодействовать в кадре? Будут ли они иметь одинаковое значение? Если нет, то насколько подчиненными будут другие предметы? Где они будут располагаться по отношению к доминирующему предмету? Будут ли они все расположены в одной фокальной плоскости? Будут ли они все в фокусе? Как заставить их координировать свои движения, жесты и выражение лица? И какое фокусное расстояние вы будете использовать, чтобы снимать их без искажений?

Это то, о чем я думаю, когда ищу возможность сфотографироваться. Я ищу композиционные переменные, чтобы разбить пустое пространство на заднем плане. Я ищу плотные узоры, яркие цветовые блоки и реквизиты окружающей среды, которые можно использовать для взаимодействия с субъектом. Я ищу случайные встречи привлекательных находок, чтобы интегрировать объект в фон. При этом я выполняю цель фотографии, которая заключается в том, чтобы заполнить пустое пространство композиционными переменными, которые оптимизируют объект и фон в документации.

Зная цель фотографии, я рассматриваю каждую фотопрогулку как момент решения проблемы. То, как и что я фотографирую, полностью зависит от моей методологии решения проблем. При этом я рассматриваю свой объект как константу в уравнении фотосъемки. В свою очередь, я рассматриваю все остальное как потенциальные композиционные переменные, необходимые для заполнения пробела. Итак, приравнивая мою константу к моим переменным посредством операции оптимального кадрирования на идеальном расстоянии съемки и фокусном расстоянии, в результате получаются фотографии, которые выглядят завершенными.

Fujifilm Superia Venus 800 с фокусным расстоянием 35 мм. Фотобомбардировщик был отредактирован рядом с левой рукой Лидии в центре справа от изображения.

Fujifilm Superia Venus 800 с фокусным расстоянием 35 мм — взаимодействует с окружающей средой, сидя на выступе.

Fujifilm Superia Venus 800 с фокусным расстоянием 35 мм

Fujifilm Superia Venus 800 с фокусным расстоянием 35 мм — взаимодействие с окружающей средой путем стояния на ручном вилочном погрузчике. Композиционные переменные включают текстурированную стену и деревянные поддоны.

Fujifilm Superia Venus 800 с фокусным расстоянием 35 мм. Композиционные переменные включают ржавчину на двери.

Fujifilm Superia Venus 800 с фокусным расстоянием 35 мм — взаимодействие с окружающей средой путем подъема по лестнице. Композиционные переменные включают текстуру и цвет на стене.

Хорошая композиция может выполнять не только заполнение пустого пространства, но и другую функцию. Он может управлять движением глаз зрителя. Он может подчеркнуть визуальное повествование, чтобы вызвать желаемую реакцию у зрителя. Или вообще, это может просто сделать фото более эстетичным. Лично я считаю, что хорошая композиция может дать фотографии впечатление цели. По большей части это приглашение испытать, что делает объект в момент документирования — по крайней мере, для моих фотографий.

В любом случае, попробуйте посмотреть на фотографию как на процесс решения проблем. Воспринимайте каждую возможность фотографирования как шанс заполнить пустое пространство композиционными переменными (цветами и формами), чтобы оптимизировать кадрирование ради получения наилучшей фотографии.

Fujifilm Superia Venus 800 с фокусным расстоянием 35 мм. Композиционные переменные включают текстурированную стену, плитку, кирпичи и деревянные поддоны.

X 1 2 разложить: Mathway | Популярные задачи

23 6 Решить для ? cos(x)=1/2 7 Найти x sin(x)=-1/2 8 Преобразование градусов в радианы 225 9 Решить для ? cos(x)=(квадратный корень из 2)/2 10 Найти x cos(x)=(квадратный корень из 3)/2 11 Найти x sin(x)=(квадратный корень из 3)/2 92=9 14 Преобразование градусов в радианы 120 градусов 15 Преобразование градусов в радианы 180 16 Найти точное значение желтовато-коричневый(195) 92-4 38 Найти точное значение грех(255) 39 Оценить лог база 27 из 36 40 Преобразовать из радианов в градусы 2 шт. 92-3sin(x)+1=0 43 Найти x tan(x)+ квадратный корень из 3=0 44 Найти x sin(2x)+cos(x)=0 45 Упростить (1-cos(x))(1+cos(x)) 92=25 59 График f(x)=- натуральный логарифм x-1+3 60 Найдите значение с помощью единичного круга угловой синус(-1/2) 61 Найти домен квадратный корень из 36-4x^2 92=0 66 Найти x cos(2x)=(квадратный корень из 2)/2 67 График у=3 68 График f(x)=- логарифмическая база 3 x-1+3 92 71 Найти x квадратный корень из x+4+ квадратный корень из x-1=5 72 Решить для ? cos(2x)=-1/2 73 Найти x логарифмическая база x из 16=4 9х 75 Упростить (cos(x))/(1-sin(x))+(1-sin(x))/(cos(x)) 76 Упростить сек(х)sin(х) 77 Упростить кубический корень из 24 кубический корень из 18 92=0 96 Найти x 3x+2=(5x-11)/(8г) 97 Решить для ? sin(2x)=-1/2 98 Найти x (2x-1)/(x+2)=4/5 92+n-72)=1/(n+9)

Калькулятор — расширить ((x+1)(x+2)) — Solumaths

Expand, расчет онлайн

Резюме:

Калькулятор способен расширить алгебраическое выражение онлайн и удалить ненужные скобки.

развернуть онлайн


Описание:

В математике до расширяют выражение или до расширяют произведение которая превращается в алгебраическую сумму. Этот калькулятор позволяет расширить все формы алгебраических выражения онлайн , также помогает рассчитать специальные разложения онлайн (разность квадратов, тождество квадрата суммы и тождество квадрата разности). Для простых расширений калькулятор дает шаги расчета.

Расширение алгебраических выражений

Калькулятор расширения позволяет онлайн расширять все формы математических выражений, выражение может быть буквенно-цифровым, т.е. он может содержать цифры и буквы: 93`

Обратите внимание, что результат не возвращается как самое простое выражение, чтобы можно было следовать шагам вычислений. Чтобы упростить результаты, просто используйте функцию сокращения. 2`. 92`

Рассчитать онлайн с помощью Expand (расширить калькулятор)

См. также

Список связанных калькуляторов:

  • Расчет ежемесячных платежей по страхованию кредита : кредит_страхование. Калькулятор ежемесячных платежей по кредитному страхованию: кредит под залог недвижимости, потребительский кредит и другие виды кредита.
  • Алгебра калькулятор: калькулятор. Калькулятор, позволяющий производить алгебраические вычисления, комбинируя операции с буквами и цифрами, а также указывать этапы вычислений.
  • Калькулятор упрощения surds :simple_surd. Онлайн-калькулятор, который позволяет производить расчеты в точной форме с квадратными корнями: сумма, произведение, разность, отношение.
  • Тригонометрический калькулятор: simple_trig. Калькулятор, который использует тригонометрическую формулу для упрощения тригонометрического выражения.
  • Список вычислений, применимых к алгебраическому выражению: см._возможные_вычисления. Возвращает список вычислений, которые можно выполнить над алгебраическим выражением.
  • Расчет биномиальных коэффициентов: binomial_coefficient. Калькулятор биномиального коэффициента, который позволяет вычислить биномиальный коэффициент из двух целых чисел.
  • Калькулятор разложения на частичные дроби: partial_fraction_decomposition. Калькулятор позволяет разбить рациональную дробь на простые элементы.
  • Калькулятор производных: производная. Калькулятор производной позволяет пошагово вычислить производную функции по переменной.
  • Калькулятор расширения Тейлора: taylor_series_expansion. Калькулятор ряда Тейлора позволяет вычислить разложение Тейлора функции.
  • Логарифмическое расширение: expand_log. Калькулятор позволяет получить логарифмическое расширение выражения.
  • Тригонометрическое расширение: expand_trigo. Калькулятор позволяет получить тригонометрическое разложение выражения.
  • Расширить калькулятор : развернуть. Калькулятор умеет расширять алгебраическое выражение онлайн и удалять ненужные скобки.
  • Расширьте и упростите алгебраическое выражение онлайн: expand_and_simplify. Онлайн-калькулятор, позволяющий расширить и сократить алгебраическое выражение.
  • Калькулятор факторинга: коэффициент. Калькулятор факторинга позволяет факторизовать алгебраическое выражение онлайн с шагом.
  • Генератор решенных математических упражнений : Exercise_generator. Возвращает список утверждений математических упражнений и их решений, которые могут использоваться учителями для подготовки тестов и викторин.
  • Интегральный калькулятор: интегральный. Калькулятор интегралов вычисляет онлайн интеграл функции между двумя значениями, результат выдается в точном или приближенном виде.
  • Калькулятор неопределенного интеграла: первообразная. Калькулятор первообразной позволяет рассчитать первообразную онлайн с подробностями и шагами расчета.
  • Калькулятор лимита: лимит. Калькулятор лимита позволяет рассчитать лимит функции с подробным описанием и шагами расчета.
  • Тригонометрическая линеаризация : linearization_trigo. Калькулятор, позволяющий линеаризовать тригонометрическое выражение.
  • Расчет ежемесячных платежей по кредиту: month_loan. Калькулятор ежемесячного платежа по кредиту: жилищный кредит, потребительский кредит и другие виды кредита.

Как называются эти фигуры: Как называются эти фигуры? — презентация по Геометрии

Как называются эти фигуры ? презентация, доклад

ThePresentationru

  • Регистрация |
  • Вход
  • Загрузить
  • Главная
  • Разное
  • Дизайн
  • Бизнес и предпринимательство
  • Аналитика
  • Образование
  • Развлечения
  • Красота и здоровье
  • Финансы
  • Государство
  • Путешествия
  • Спорт
  • Недвижимость
  • Армия
  • Графика
  • Культурология
  • Еда и кулинария
  • Лингвистика
  • Английский язык
  • Астрономия
  • Алгебра
  • Биология
  • География
  • Геометрия
  • Детские презентации
  • Информатика
  • История
  • Литература
  • Маркетинг
  • Математика
  • Медицина
  • Менеджмент
  • Музыка
  • МХК
  • Немецкий язык
  • ОБЖ
  • Обществознание
  • Окружающий мир
  • Педагогика
  • Русский язык
  • Страхование
  • Технология
  • Физика
  • Философия
  • Химия
  • Шаблоны, картинки для презентаций
  • Экология
  • Экономика
  • Юриспруденция

contents-content» data-classactive=»contents-heading_expand»> Содержание

  • 1. Как называются эти фигуры ?
  • 2. Как можно называть эту фигуру ?
  • 3. Гладкова Ольга
  • 4. Сколько на чертеже
  • 6. Овал
  • 7. Треугольник
  • 8. Квадрат Ромб
  • 9. Гладкова Ольга
  • 10. Прямоугольник
  • 11. Гладкова Ольга
  • 12. Гладкова Ольга
  • 13. Гладкова Ольга
  • 14. Гладкова Ольга
  • 15. Гладкова Ольга
  • 16. Гладкова Ольга
  • 17. Молодцы Гладкова

Как можно называть эту фигуру ? Четырёхугольник Многоугольник Квадрат Прямоугольник ✓ ✓ ✓ Гладкова Ольга Владимировна учитель начальных классов Гимназии № 54 город

Слайд 1Как называются эти фигуры ?


Гладкова Ольга

Владимировна учитель начальных классов Гимназии № 54 город Краснодар


Слайд 2Как можно называть эту фигуру ?

Четырёхугольник
Многоугольник
Квадрат
Прямоугольник



Гладкова Ольга Владимировна учитель начальных классов Гимназии № 54 город Краснодар


Слайд 3 Гладкова Ольга Владимировна учитель начальных классов

Гимназии № 54 город Краснодар

Что такое квадрат ?

Квадрат – это прямоугольник у которого все стороны равны


Сколько на чертеже прямоугольников?
Гладкова Ольга Владимировна

учитель начальных классов Гимназии № 54 город Краснодар


Какие геометрические фигуры вы узнали на рисунке?

Гладкова Ольга Владимировна учитель начальных классов Гимназии № 54 город Краснодар


Гладкова Ольга Владимировна учитель начальных классов

Гимназии № 54 город Краснодар


Гладкова Ольга Владимировна учитель начальных классов

Гимназии № 54 город Краснодар


Гладкова Ольга Владимировна учитель начальных классов

Гимназии № 54 город Краснодар


Слайд 9 Гладкова Ольга Владимировна учитель начальных классов

Гимназии № 54 город Краснодар

Трапеция


Гладкова Ольга Владимировна учитель начальных классов

Гимназии № 54 город Краснодар


Слайд 11 Гладкова Ольга Владимировна учитель начальных классов

Гимназии № 54 город Краснодар

Круг


Слайд 12 Гладкова Ольга Владимировна учитель начальных классов

Гимназии № 54 город Краснодар

Назовите формулу нахождения периметра прямоугольника

a + b + a + b

(a + b) ● 2

a ● b




Слайд 13 Гладкова Ольга Владимировна учитель начальных классов

Гимназии № 54 город Краснодар

Как называются эти углы?

А

В

С


Слайд 14 Гладкова Ольга Владимировна учитель начальных классов

Гимназии № 54 город Краснодар

МОЛОДЕЦ !


Слайд 15 Гладкова Ольга Владимировна учитель начальных классов

Гимназии № 54 город Краснодар

ПОДУМАЙ !


Слайд 16 Гладкова Ольга Владимировна учитель начальных классов

Гимназии № 54 город Краснодар

Покажи у фигуры прямой угол

D

А

С

В


Слайд 17Молодцы


Гладкова Ольга Владимировна учитель начальных классов

Гимназии № 54 город Краснодар


Скачать презентацию

Обратная связь

Если не удалось найти и скачать презентацию, Вы можете заказать его на нашем сайте. Мы постараемся найти нужный Вам материал и отправим по электронной почте. Не стесняйтесь обращаться к нам, если у вас возникли вопросы или пожелания:

Email: Нажмите что бы посмотреть 

Что такое ThePresentation.ru?

Это сайт презентаций, докладов, проектов, шаблонов в формате PowerPoint. Мы помогаем школьникам, студентам, учителям, преподавателям хранить и обмениваться учебными материалами с другими пользователями.


Для правообладателей

Как называются пространственные геометрические фигуры?

содержание

Что такое пространственные геометрические фигуры?

Примеры пространственных фигур: пирамида, призма, куб, сфера, цилиндр и др.

Сколько пространственных геометрических фигур?

Пространственные геометрические фигуры# Пространственная геометрия изучает различные геометрические тела, среди основных у нас есть: цилиндр, куб, конус, сфера, параллелепипед и пирамида.

Как называются все геометрические фигуры?

Кроме того, у каждого из них есть несколько сторон.

  • Треугольник — 3 стороны.
  • Четырехугольник – 4 стороны.
  • Пентагон – 5 сторон.
  • Шестиугольник — 6 сторон.
  • Гептагон – 7 сторон.
  • Октагон – 8 сторон.
  • Эннеагон – 9 сторон.
  • Декагон – 10 сторон.

Что такое космические фигуры?

Пространственные фигуры необходимо определять в трехмерных пространствах, так как они имеют глубину в дополнение к длине и ширине. Кубы, призмы, цилиндры, конусы и сферы, например, являются фигурами, которые могут быть определены только в трехмерном пространстве.

Что такое пространственные геометрические фигуры 2 основного года?

Космические геометрические фигуры, называемые также геометрическими телами, — это те, которые имеют три измерения: длину, ширину и глубину.

Какие плоские фигуры образуют пространственную фигуру?

Поэтому любая фигура, для построения и определения которой необходимы три измерения, называется пространственной геометрической фигурой. Примеры пространственных фигур: куб, призма, параллелепипед, пирамида, конус, цилиндр, сфера и др.

Что касается названия шестиугольной призмы и пространственной формы?

Шестиугольная призма: основание образовано шестиугольником. Семиугольная призма: основание образовано семиугольником.

Какие 4 основные геометрические фигуры?

Треугольники, квадраты, прямоугольники и пятиугольники являются примерами плоских геометрических фигур. Неплоские (пространственные) геометрические фигуры имеют ширину, длину и толщину, будучи трехмерными. Эти формы делятся на многогранники и не-многогранники (круглые тела).

Какие 4 геометрические фигуры?

плоские геометрические фигуры

Количество сегментов линииКлассификация
4четырехугольник
5пятиугольник
6Гексагоно
7Семиугольник

Что такое плоские геометрические фигуры?

Основными плоскими фигурами являются треугольник, круг, квадрат, прямоугольник, ромб и трапеция, и каждая из них имеет формулу вычисления площади. Стоит отметить, что область изучается в планиметрии, геометрии для двухмерных объектов.

Как сделать пространственную геометрию?

Тетраэдр – это правильная пирамида с четырьмя конгруэнтными гранями, причем треугольники граней равносторонние. Поскольку площадь равностороннего треугольника определяется как Ab = (a²*√3)/4, а высота определяется как h = (a*√6)/3, мы имеем: V = Ab*h/3 = (а²* √3)*(а*√6)/3*4*3 = а³*√2/12.

Чем отличаются плоские геометрические фигуры от пространственных?

Планиметрия — это изучение фигур в двух измерениях, таких как квадраты, круги, прямоугольники и треугольники. В то время как пространственная геометрия изучает фигуры в трех измерениях, то есть кубы, сферы, параллелепипеды и пирамиды.

Что такое 6 пространственных геометрических фигур?

Особенности пространственной геометрии

  • призма.
  • Куб.
  • брусчатка.
  • пирамида.
  • конус.
  • цилиндр.
  • мяч.

Является ли это пространственной геометрической фигурой, у которой все грани квадратные?

Также известный как шестигранник, куб представляет собой геометрическое тело, все грани которого образованы квадратами. У него 6 граней, 12 ребер и 8 вершин. Куб представляет собой шестигранный многогранник, все грани которого квадратные.

Что такое 4-летние пространственные геометрические фигуры?

ПРОСТРАНСТВЕННЫЕ ГЕОМЕТРИЧЕСКИЕ ФИГУРЫ ИЛИ ГЕОМЕТРИЧЕСКИЕ ТЕЛА ЯВЛЯЮТСЯ ТЕ, КОТОРЫЕ ПРОИСХОДЯТ В ПРОСТРАНСТВЕ, В СВЯЗИ С ИХ ТРЕХМЕРНОСТЬЮ, Т.Е. ИМЕЮТ ТРИ ИЗМЕРЕНИЯ (ВЫСОТУ, ШИРИНУ И ДЛИНУ).

Какие из приведенных ниже пространственных геометрических фигур являются многогранниками?

На основе примитивных элементов разрабатываются геометрические тела, главными из которых являются многогранники: параллелепипеды, кубы и другие призмы, помимо так называемых платоновских тел; и круглые тела: конус, цилиндр и сфера.

Являются ли они неплоскими космическими фигурами?

Неплоские или пространственные геометрические фигуры — это фигуры, расположенные в пространстве. Эти фигуры расположены в трех измерениях: длина, ширина и высота.

Как называются все призмы?

четырехугольная призма: имеет каждое из оснований в форме четырехугольника; пятиугольная призма: имеет каждое из оснований в форме пятиугольника; шестиугольная призма: имеет каждое из оснований в форме шестиугольника; восьмиугольная призма: имеет каждое из оснований в форме восьмиугольника.

Что такое специальные призмы?

3. Пирамиды и специальные призмы. Особой призмой, например, является куб: это призма с квадратными основаниями и равными боковыми гранями, то есть фигура имеет шесть равных граней, образованных квадратами.

Как называется геометрическая фигура, имеющая 8 граней?

Октаэдр имеет 8 конгруэнтных треугольных граней и 6 конгруэнтных тетраэдрических углов.

Что такое пространственные геометрические фигуры и как они образуются?

Пространственные геометрические фигуры – это те, которые имеют три измерения: длину, высоту и ширину. Эти фигуры делятся на две группы: круглые тела (ограниченные какой-либо округлой поверхностью) и многогранники (поверхности, ограниченные плоскими геометрическими фигурами).

Как называются все многоугольники?

Поэтому по отношению к числу сторон многоугольникам можно давать следующие названия:

  • 3 стороны → треугольник или трехугольник.
  • 4 стороны → квадрат или четырехугольник.
  • 5 сторон → пятиугольник или пятиугольник.
  • 6 сторон → шестиугольник или шестигранник.
  • 7 сторон → семиугольник или семиугольник.
  • 8 сторон → восьмиугольник или восьмиугольник.

Что такое плоские и неплоские геометрические фигуры?

Неплоские геометрические фигуры (куб, прямоугольный блок, пирамида, конус, цилиндр и сфера): распознавание и характеристика. Плоские геометрические фигуры (круг, квадрат, прямоугольник и треугольник): распознавание и характеристика.

Что такое плоские фигуры?

Мы называем плоской фигурой любую фигуру, имеющую два измерения. Нас окружают плоские фигуры, такие как квадраты, круги, треугольники и другие формы. Плоские фигуры имеют два измерения. Плоские фигуры присутствуют во все времена в быту.

Сколько и какие полигоны?

Посмотрите названия различных типов многоугольников в зависимости от количества их сторон.

Типы полигонов.

количество сторонимя
4четырехугольник
5пятиугольник
6Гексагоно
7Семиугольник

Как называется фигура, у которой 5 сторон?

J. 2 полигона

четырехугольник44
пятиугольник55
Гексагоно66
Семиугольник77
восьмиугольник88

Сколько граней у призмы?

У него 8 вершин, 12 ребер и 6 граней, две из которых являются основаниями, а остальные прямоугольными.

Сколько граней у параллелепипеда?

Параллелепипед – это геометрическое тело, грани которого образованы параллелограммами. Он состоит из 8 вершин, 12 ребер и 6 граней. Он классифицируется как прямой, если его края перпендикулярны, и как косой, если его края не перпендикулярны.

Что такое круглое тело?

Цилиндр, конус и сфера являются круглыми телами. Мы называем круглыми телами геометрические тела, имеющие криволинейные поверхности. Они также известны как тела вращения, потому что они построены путем вращения плоской фигуры.

Кто создал геометрию?

Евклид, великий математик и писатель, жил, вероятно, в XNUMX веке до нашей эры и считается отцом геометрии. Он первым собрал всю геометрию в единое произведение под названием «Элементы». Этот математик основывал планиметрию на пяти постулатах.

Что еще относится к пространственной геометрии?

Наиболее часто повторяющейся темой в тестах, связанных с пространственной геометрией, является расчет объема геометрических тел. Помимо расчета объема часто возникают вопросы по идентификации геометрических тел, их характеристикам и свойствам.

Что такое геометрические объекты?

Геометрические тела — это трехмерные объекты, заданные в пространстве. Некоторые примеры геометрических тел: кубы, пирамиды, призмы, цилиндры и сферы. Совокупность всех геометрических тел принято делить на три большие группы: многогранники, круглые тела и другие.

Каковы пространственные различия?

Таким образом, пространственная дифференциация была бы просто географическим соответствием, т. е. различием, существенным, имманентным условием, выраженным в пространстве, так как она видна как в природных условиях (почва, растительность, климат), так и в аспектах социальных (экономика, политика, культура).

Где используется пространственная геометрия?

Куб, цилиндр, конус, пирамиды и другие являются объектами изучения космической геометрии. С помощью пространственной геометрии можно обнаружить характеристики и свойства геометрических тел, а также разработать формулы для расчета объема и площади этих тел.

Чем отличаются плоские фигуры от пространственных?

Планиметрия — это изучение фигур в двух измерениях, таких как квадраты, круги, прямоугольники и треугольники. В то время как пространственная геометрия изучает фигуры в трех измерениях, то есть кубы, сферы, параллелепипеды и пирамиды.

Какую группу я могу рассматривать как плоские фигуры и пространственные геометрические фигуры?

Аннотация о плоских фигурах

  • Это фигуры, имеющие два измерения.
  • Основные из них: треугольник; площадь;
  • Плоские фигуры всегда двухмерны. Когда есть трехмерные фигуры, они известны как пространственные.
  • На основных плоских рисунках у каждого из них есть определенная формула для расчета его площади.

Как сделать пространственную геометрию?

Тетраэдр – это правильная пирамида с четырьмя конгруэнтными гранями, причем треугольники граней равносторонние. Поскольку площадь равностороннего треугольника определяется как Ab = (a²*√3)/4, а высота определяется как h = (a*√6)/3, мы имеем: V = Ab*h/3 = (а²* √3)*(а*√6)/3*4*3 = а³*√2/12.

Определение, типы, список, решенные примеры, факты

Что такое формы?

В геометрии форма может быть определена как форма объекта или его контур, внешняя граница или внешняя поверхность.

Все, что мы видим в окружающем нас мире, имеет форму. Мы можем найти различные основные формы, такие как двухмерный квадрат, прямоугольник и овал или трехмерную прямоугольную призму, цилиндр и сферу, в объектах, которые мы видим вокруг нас. Эти геометрические формы появляются в объектах, которые мы видим, таких как кредитные карты, банкноты и монеты, перстни, фоторамки, доски для игры в дартс, хижины, окна, волшебные палочки, высокие здания, цветочные горшки, игрушечные поезда и воздушные шары.

Связанные игры


Различные типы фигур

Фигуры можно разделить на открытые и замкнутые.

В геометрии незамкнутая форма может быть определена как форма или фигура, сегменты линий и/или кривые 
которых не пересекаются. Они не начинаются и не заканчиваются в одной и той же точке.
В геометрии замкнутая фигура может быть определена как замкнутая форма или фигура, сегменты линий и/или кривые которых соединены или пересекаются. Они начинаются и заканчиваются в одной и той же точке.

Замкнутые геометрические формы можно разделить на две широкие категории, а именно двухмерные формы и трехмерные формы.

Плоская двухмерная форма. Трехмерная форма — это твердая форма.
Он имеет два измерения, то есть длину и ширину. Он имеет два измерения: длину, ширину и глубину.

Вот список двухмерных или двухмерных фигур с их названиями и изображениями:

Двумерные геометрические формы

Здесь Список 3-D или трехмерные формы с их названиями и картинками:

Трехэ.

Цвет, общий размер и ориентация, называемые неопределяющими атрибутами двухмерной или трехмерной формы, никак не определяют форму и не влияют на нее. Эти атрибуты могут изменяться без какого-либо влияния на форму.

С другой стороны, определение таких атрибутов, как количество сторон (параллельных или непараллельных, прямых или изогнутых), вершин, ребер и граней формы, открытость формы или замкнутость, а также угловые меры определяют форма двухмерного или трехмерного объекта. Любое изменение этих определяющих атрибутов изменит форму.

Связанные рабочие листы

Решенные примеры фигур

Пример 1: Назовите фигуры.

  1. Многоугольник с 6 сторонами.
  2. Контур двери.
  3. При складывании квадрата угол в угол.
  4. Квадрат и треугольник поверх него.

Решение:

  1. Шестиугольник
  2. Прямоугольник или четырехугольник
  3. Треугольник
  4. Пентагон

Пример 2: Классифицируйте данные буквы как открытые или закрытые.

C, D, L, M, O, S, U, V, Z

Решение:

Незамкнутая форма: C, L, M, S, U, V, Z

Замкнутая форма: D, O

Пример 3: Определите объемную форму данных объектов.

  1. Глобус
  2. Книга
  3. Банка для холодных напитков
  4. Кости

Решение:

  1. Сфера
  2. Прямоугольный
  3. Цилиндр
  4. Куб

Пример 4: Почему луна в форме полумесяца не является многоугольником?

Решение:  

Луна в форме полумесяца не является многоугольником, поскольку имеет изогнутые линии.

Практические задачи на фигурах

1

Как называется 8-сторонний многоугольник?

шестиугольник

семиугольник

восьмиугольник

четырехугольник

Правильный ответ: восьмиугольник
Многоугольник с 8 сторонами известен как восьмиугольник.

2

Сколько измерений имеет твердое тело?

1

2

3

зависит от формы

Правильный ответ: 3
Все объемные формы являются трехмерными.

3

Какое из следующих утверждений неверно?

замкнутые формы могут иметь только прямые стороны.

замкнутые формы имеют определенную площадь.

начальная и конечная точки замкнутой формы совпадают.

Начальная и конечная точки открытой формы разные.

Правильный ответ: замкнутые формы могут иметь только прямые стороны.
Замкнутые фигуры — это фигуры, у которых начальная и конечная точки совпадают. Не обязательно, чтобы он образовывался только прямыми сторонами.

Как называются 2D- и 3D-формы?

Мы объясняем, чем отличаются двухмерные и трехмерные фигуры, когда детей младшего школьного возраста учат называть их и сортировать фигуры в соответствии с их свойствами, а также когда они учатся идентифицировать и рисовать свои собственные сети трехмерных фигур.

или зарегистрируйтесь, чтобы добавить к своим сохраненным ресурсам

Во время обучения в начальной школе дети будут изучать различные 2D (двухмерные) и 3D (трехмерные) формы.

Первое, что им нужно усвоить, это разницу между 2D и 3D формами. Учителя часто говорят о том, что 2D-формы «плоские», а 3D-формы не . Ожидается, что дети смогут назвать эти фигуры, а также обсудить свойства этих фигур.

2D-фигуры
Круг Square
Triangle Rectangle
Pentagon Hexagon
Octagon Нонагон

Трехмерные формы
Cube Cuboid
Sphere Square-based pyramid
Cylinder Triangular prism
Пятиугольная пирамида Шестиугольная призма

2D- и 3D-фигуры в KS1

В 1-й год дети должны уметь распознавать и называть:

  • 2D-фигуры, включая прямоугольники, квадраты, круги и треугольники
  • 3D-фигуры, включая кубы , пирамиды и сферы
  • сортировать, создавать и описывать распространенные 2D- и 3D-формы

В 2-м классе дети должны уметь:

  • определять и описывать свойства 2D-фигур, включая симметрию и линейную симметрию
  • идентифицировать и описывать 3D-формы, включая количество ребер, вершин и граней
  • идентифицировать 2D-фигуры на поверхности 3D-фигур
  • сравнивать и сортировать распространенные 2D- и 3D-фигуры
  • понимать, что прямой угол — это четверть оборота и знать, по часовой стрелке или против часовой стрелки

 

2D и 3D фигуры в KS2

В Year 3 дети должны:

  • рисовать 2D фигуры и делать 3D фигуры
  • определять прямые углы и знать, что два из них делают пол-оборота, три — на три четверти, а четыре — полный оборот необходимо:

    • сравнивать четырехугольники и треугольники по их свойствам и размерам
    • определять острые и тупые углы, сравнивать и упорядочивать углы
    • определять линии симметрии в двумерных фигурах

    В 5 классе дети должны:

    • определять трехмерные фигуры из двухмерных представлений
    • оценивать и сравнивать острые, тупые и обратные углы
    • рисовать заданные углы между правильными и неправильными многоугольниками

    Дети, изучающие в 6 классе , должны:

    • классифицировать 2D и 3D формы, рассказывая о параллельных и перпендикулярных ребрах и гранях
    • рисовать двухмерные фигуры, используя заданные размеры и углы
    • распознавать, описывать и строить простые трехмерные фигуры, включая создание сетей
    • находить неизвестные углы в треугольниках, четырехугольниках и правильных многоугольниках окружности и знать, что диаметр в два раза больше радиуса

    Другие термины 2D и 3D форм, используемые в KS2

    Равносторонний треугольник: Треугольник с 3 равными сторонами и 3 равными углами. Равнобедренный треугольник:  Треугольник с 2 равными сторонами.
    Разносторонний треугольник:  Треугольник с 3 неравными сторонами. Прямоугольный треугольник:  Треугольник с прямым углом.
    Четырехугольник:  Четырехсторонняя форма. Ромб:  Четырехугольник, у которого обе пары или противоположные стороны параллельны и все стороны имеют одинаковую длину. (Отличается от квадрата тем, что углы НЕ 90˚).
    Трапеция:  Четырехугольник с одной парой параллельных сторон. Параллелограмм:  Четырехугольник, у которого обе пары противоположных сторон параллельны и обе пары имеют одинаковую длину.
    Воздушный змей : Четырехугольник с двумя парами сторон одинаковой длины.

    Рт по математике онлайн: Карта сайта | математики.бел

    Карта сайта | математики.бел

    1. Главная
    2. Тесты
    • Главная

      Креатив обычно правомочен. Узнавание бренда, согласно Ф.Котлеру, регулярно стабилизирует потребительский рекламный клаттер, размещаясь во всех медиа. Формат события программирует общественный бренд, учитывая современные тенденции.

      • Информация
        • Информация
        • Новости
        • Статьи
        • Акции
      • Отзывы
      • Акции
    • О нас
      • О нас
      • Полезные ссылки
      • Сотрудники
      • Отзывы
      • Вопрос ответ
      • Фотографии
      • История
      • Свидетельства и сертификаты
    • Каталог
    • Информация
    • Репетиторы
      • Школьникам математика
        • Подготовка к ЦТ, ЦЭ
        • Подготовка к ЕГЭ
        • Поступающим в зарубежные ВУЗы
        • 11 класс
        • 10 класс
        • 9 класс
        • 8 класс
        • 7 класс
        • 6 класс
      • Школьникам физика
        • Подготовка к ЦТ
        • 11 класс
        • 10 класс
        • 9 класс
        • 8 класс
      • Репетитор для студентов
        • Высшая математика
        • Теория вероятностей
        • Математическое программирование
        • Статистика
        • Физика
        • Биофизика
        • Бизнес — Статистика
        • Биостатистика
        • Математическая статистика для медиков
        • Математическая статистика для психологов
        • Финансовая математика
      • Онлайн репетитор
        • Математика
        • Высшая математика
        • Статистика
      • Английский язык
    • Тесты
      • Тесты для подготовки к ЦТ, ЦЭ 2023 по математике
      • Тесты по Алгебре
      • Тесты по Геометрии
    • Теория
    • Контакты

    Курсы по математике онлайн | Подготовка к ЦТ, ЦЭ

    Математика – одна из главных дисциплин, изучаемых в средних общеобразовательных учебных заведениях, колледжах, лицеях и вузах. Эта наука применяется в качестве прикладной в физике, химии, программировании, астрономии, лингвистике. Школьникам приходится сдавать математику в виде ЦТ – на территории Республики Беларусь или ЕГЭ (ОГЭ) – в России. Это необходимо для поступления в институты, университеты и иные учебные заведения. Уроки математики в режиме онлайн проводится в Центре Анатолия Лазуркина преподавателями со стажем не менее 10-20 лет.

     

    Онлайн курс математики. Особенности программы

    За 1-2 месяца до экзаменов школьники начинают усиленно готовиться к тестам. В ряде случаев приходится прибегать к помощи со стороны: обращаться к репетиторам-математикам, посещать уроки. Это приводит к потере денег и времени, которое лучше потратить на подготовку к испытаниям.

    Обучение online позволяет организовать учебный процесс так, что будущему студенту не нужно выходить из дома и посещать класс. Для изучения предметов используется компьютер, абитуриенту нужен лишь доступ в интернет.

    Преподаватели проводят онлайн-лекции, вебинары, предварительное тестирование. Каждый школьник получает индивидуальные задания в зависимости от его фактической подготовки, решает примеры из школьной программы. Все это происходит в привычной для обучающегося обстановке. Наши репетиторы математики оказывают помощь в решении любой задачи в режиме онлайн, проводят консультации в удобное для ученика время.

     

    Методика Анатолия Лазуркина для занятий математикой онлайн

    Уникальная программа прошла испытание в течение 20 лет. Она дает возможность в короткий срок изучить сложный предмет, не занимаясь нудным заучиванием теорем, формул, правил. В основе – особенные «зацепки для памяти», которые позволяют школьнику лучше запомнить сложные математические понятия. В процессе обучения абитуриент получает бесплатные материалы для самоподготовки по нашему методу.

    Постоянно проводимые тесты выявляют слабые места в уровне подготовки обучающегося. Преподаватель в этом случае начинает делать упор на тех разделах, которые вызывают у школьника большие проблемы. Проводится скрупулезная работа с каждым учеником. В результате эффективность обучения увеличивается по сравнению с обычными методами. По нашей статистике каждый год большинство наших учеников успешно сдают тесты и получают высокий балл.

     

    Как проводятся занятия

    Математика, изучаемая в средних учебных заведениях, состоит из разделов:

    • алгебра;
    • геометрия;
    • математический анализ (начало).

    Подготовка к вступительным экзаменам в ВУЗ требует отличного знания основных формул, навыков решения задач.

    Мы проводим занятия в течение всего учебного года. Главная цель – подготовить абитуриента к решению любой задачи, тестов ЦТ. Для тех, кто хорошо владеет предметом, достаточно полугодовой программы обучения на наших курсах.

    Индивидуальные планы обучения преподаватель создает в зависимости от уровня знания обучаемого. Для этого проводится предварительное собеседование и тест.

    Цель обучения – научить школьника уверенно справляться с ЦТ. Уровень сложности наших тестов соответствует теми, которые ученику придется решать во время централизованного тестирования на территории Республики Беларусь. Все ЦТ разделено на 2 части:

    1. Первая (A). Состоит из 18 заданий, к каждому из них предложено 5 вариантов ответа. Они размещены в произвольном порядке. Самое первое задание может оказаться наиболее трудным, а последнее – простым. В заданиях этой части правильный ответ может быть лишь один. На выполнение первой части тратится в среднем до 30 минут.
    2. Вторая (В). Состоит из 11 вопросов. Для правильного решения тестовых заданий из второй части потребуется не менее 60 минут. Ответы на задания лучше всего записывать в виде чисел. Важно сначала ответить на те вопросы, которые абитуриенту известны. При необходимости, можно использовать калькулятор.

    Главное правило: чем больше учащийся решил заданий до теста, тем увереннее он будет себя чувствовать во время испытаний. Знание теоретической части позволит справиться даже с самыми сложными примерами. Одна из наших целей – развить у ученика способности по самостоятельному решению любых математических задач.

    Все преподаватели Центра Лазуркина обладают большим опытом. Они могут не только научить решать стандартные задачи математики, но также помогут одолеть более трудные. Такие навыки пригодятся и после ЦТ – во время обучения в вузах или колледжах.

    Мы используем для обучения не только литературу, но и интерактивные материалы, которые позволяют лучше и быстрее усвоить предмет. Все это – часть эффективной методики Лазуркина.

     

    Как записаться на курс обучения

    Позвоните по телефону или заполните заявку. Или посетите нашу страницу в Vkontakte. Наши консультанты расскажут о времени начала обучения, стоимости обучения, способах оплаты. После записи мы обязательно напомним вам о дате первого урока. Очное обучение проводится в Минске, Витебске, Могилеве и Бресте. Для прохождения онлайн курса приезжать в Центр не нужно.

    Математическая задача: вершины RT

    Показать, что точки P1 (5,0), P2 (2,1) и P3 (4,7) являются вершинами прямоугольного треугольника.

    Правильный ответ:

    x =  0

    Пошаговое объяснение:

    x1​=5 y1​=0 x2​=2 y2​=1 x3​=4 y3​=7 a=(x1​ −x2​)2+(y1​−y2​)2

    ​=(5−2)2+(0−1)2

    ​=10

    ​≐3,1623 b=(x1​−x3​) 2+(y1​−y3​)2

    ​=(5−4)2+(0−7)2

    ​=5 2

    ​≐7,0711 c=(x2​−x3​)2+( y2​−y3​)2

    ​=(2−4)2+(1−7)2

    ​=2 10

    ​≐6,3246 x=b2−(a2+c2)=7,07112−(3,16232+6,32462)=0

    Попробуйте рассчитать с помощью нашего калькулятора треугольников.


    Нашли ошибку или неточность? Не стесняйтесь

    пишите нам

    . Спасибо!

    Советы по использованию связанных онлайн-калькуляторов

    См. также наш калькулятор прямоугольного треугольника.
    См. также наш калькулятор тригонометрического треугольника.

    Для решения этой математической задачи вам необходимо знать следующие знания:

    • геометрия
    • аналитическая геометрия
    • планиметрия
    • теорема Пифагора
    • прямоугольный треугольник
    • треугольник
    Единицы физических величин:
    • угол
    Класс задачи:
    • Практика для 14-летних
    • Старшая школа

     

    Мы рекомендуем вам посмотреть это обучающее видео по этой математической задаче: видео1   видео2

    • Вершины прямоугольного треугольника
      Покажите, что точки D(2,1), E(4,0) и F(5,7) являются вершинами прямоугольного треугольника.
    • Точки коллинеарны
      Покажите, что точки A(-1,3), B(3,2), C(11,0) лежат на одной прямой.
    • Четырехугольник 2
      Докажите, что четырехугольник с вершинами P1(0,1), P2(4,2), P3(3,6) P4(-5,4) имеет два прямоугольных треугольника.
    • Построить 8
      Построить задачу аналитической геометрии, где требуется найти вершины треугольника ABC: Вершинами этого треугольника являются точки A (1,7), B (-5,1) C (5, -11 ). В указанной задаче следует использовать понятия расстояния от точки до прямой, рати
    • Квадрат 2
      Точки D[10,-8] и B[4,5] являются противоположными вершинами квадрата ABCD. Вычислите площадь квадрата ABCD.
    • Вероятность 81637
      Мы случайным образом выбираем три разные точки из вершин правильного семиугольника и соединяем их отрезками. Вероятность того, что полученный треугольник будет равнобедренным, равна: (A) 1/3 (B) 2/5 (C) 3/5 (D) 4/7
    • Кривая и прямая
      Уравнение кривой C имеет вид y=2x² -8x+9, а уравнение прямой L равно x+ y=3 (1) Найдите координаты x точек пересечения L и C. (2) Докажите, что одна из этих точек также является стационарной точка С?
    • Три точки 2
      Три точки: A(3, 8), B(6, 2) и C(10, 2). Точка D такова, что прямая DA перпендикулярна AB, а DC параллельна AB. Вычислить координаты D.
    • Диагонали 3580
      Длина ребра куба 5см. Нарисуйте разные диагонали.
    • Гипербола
      Найдите уравнение гиперболы, проходящей через точку M [30; 24] и имеет фокусы на F1 [0; 4 кв.6], F2 [0; -4 кв.6].
    • Квадрат
      Точки A[9,9] и B[-4,1] являются смежными вершинами квадрата ABCD. Вычислите площадь квадрата ABCD.
    • Координаты 2
      Координаты вершин показанного треугольника: A(1,7), B(5,2) и C(5,7). Какова длина отрезка АВ в единицах?
    • Расстояние 4527
      Есть две точки, K и L, KL = 4 см. Проведите прямую p, проходящую через точку K и отстоящую от точки L на 4 см.
    • Координата X 81737
      В треугольнике ABC определите координаты точки B, если известно, что точки A и B лежат на прямой 3x- y-5=0, точки A и C лежат на прямой 2x+3y+4=0, точка C лежит на оси координат x, угол при вершине C прямой.
    • Перпендикуляр 28823
      Даны точки A(1,2), B(4,-2) и C(3,-2). Найдите параметрические уравнения прямой, которая: а) проходит через точку С и параллельна прямой АВ, б) проходит через точку С и перпендикулярна прямой АВ.
    • В строке
      В строке p: x = 4 + t, y = 3 + 2t, t равно R, найти точку C, которая находится на одинаковом расстоянии от точек A [1,2] и B [-1,0] .
    • Медианы и стороны
      Треугольник ABC в плоскости Oxy; координаты точек: A = 2,7 B = -4,3 C-6-1 Попробуйте вычислить длины всех медиан и всех сторон.

    Решение задач с формулой расстояния, скорости и времени — Криста Кинг Математика

    Формула, которая связывает расстояние, скорость и время

    В этом уроке показано, как найти расстояние, скорость и время, зная два из трех этих значений.

    Расстояние, скорость и время связаны уравнением

    ???\text{Расстояние}=\text{Скорость} \cdot \text{Время}???

    ???D=RT???

    Привет! Я Криста.

    Я создаю онлайн-курсы, чтобы помочь вам в учебе по математике. Читать далее.

    Давайте поговорим о единицах измерения каждого из этих значений.

    Расстояние измеряется в дюймах, футах, милях или сантиметрах, метрах, километрах и т. д.

    Время выражается в секундах, минутах, часах и т. д. миль/час или километров/час.

    Перед использованием формулы ???D=RT??? вам нужно убедиться, что ваши единицы измерения расстояния и времени совпадают с вашими ставками. Если это не так, вам нужно изменить их, чтобы вы работали с теми же единицами измерения.

    Как решать задачи на расстояние, скорость и время

    Пройти курс

    Хотите узнать больше об Алгебре 2? У меня есть пошаговый курс для этого. 🙂

    Узнать больше

    Нахождение средней скорости по заданному расстоянию и времени

    Пример

    Хизер пробежала ???56??? км в ???5??? часы. Какова была средняя скорость Хизер в км/ч?

    Воспользуемся формулой расстояния.

    ???\text{Расстояние}=\text{Скорость} \cdot \text{Время}???

    ???D=RT???

    Давайте запишем, что мы знаем.

    ???D=56??? км

    ???T=5??? hr

    Если мы подставим их в формулу расстояния, мы получим

    ???D=RT???

    ???56\текст{км} = R\cdot 5\текст{ч}???

    Теперь определите скорость.

    ???\frac{56\ \text{km}}{5\ \text{hr}} = \frac{R \cdot 5\ \text{hr}}{5\ \text{hr}}? ??

    ???R=11,2\ \frac{\text{км}}{\text{ч}}???

    Прежде чем использовать формулу D=RT, необходимо убедиться, что единицы измерения расстояния и времени совпадают с единицами измерения скорости.

    Проблемы с расстоянием, скоростью и временем с двумя людьми, уезжающими в разное время

    Пример

    Сьюзен и Бенджамину было ???60??? миль друг от друга по прямой тропе. Сьюзен пошла к Бенджамину со скоростью ???5??? миль в час в 7:30 утра. Бенджамин уехал через три часа, и они встретились на тропе в 15:30. Насколько быстр Бенджамин?

    Нам дали информацию о расстоянии, скорости и времени, поэтому мы будем использовать формулу

    ???\text{Расстояние}=\text{Скорость} \cdot \text{Время}???

    ???D=RT???

    где ???D??? пройденное расстояние, ???R??? это скорость, и ???T??? это время. Мы можем использовать индексы для создания уникальных уравнений для Сьюзен и Бенджамина.

    Сьюзен: ???D_{S} =R_{S} T_{S}???

    Бенджамин: ???D_{B} =R_{B} T_{B}???

    Мы знаем, что для того, чтобы встретиться друг с другом, они должны были преодолеть расстояние ???60??? миль между ними. Следовательно,

    ???D_{S}+D_{B}=60???

    Поскольку мы знаем, что ???D_{S}=R_{S}T_{S}??? и ???D_{B}=R_{B}T_{B}???, мы можем сделать замену в этом уравнении для их скорости и времени, вместо их расстояний.

    ???R_{S}T_{S}+R_{B}T_{B}=60???

    Проблема говорит нам, что Сьюзен шла в ???5??? миль в час, и что она шла за ???8??? часов, так как она шла с 7:30 до 15:30.

    Перечислено поставщикам за материалы проводка: Типовые ситуации по счету 60 «Расчеты с поставщиками и подрядчиками»

    Карта сайта

    Главная

    Обучение

    Библиотека

    Карта сайта

    • Главная

      Официальный сайт ДГАУ

      • Cведения об образовательной организации
      • Инклюзивное образование
      • Дополнительное профессиональное образование
        • Институт непрерывного образования (п.Персиановский)
        • Институт непрерывного образования (г.Новочеркасск)
        • Азово-Черноморский инженерный институт (г.Зерноград)
      • Новости и объявления
      • Вакансии
      • Федеральные и региональные целевые программы, государственная поддержка села
      • Информация работодателей
      • История университета в лицах
      • Перевод студентов на бюджетную форму обучения
    • Об университете

      Официальный сайт ДГАУ

      • Сведения об образовательной организации
      • История университета
      • Университет сегодня
      • Ректорат
      • Ученый совет
      • Административно-управленческий аппарат
      • Доска Почета
      • Партнеры университета
      • Информация Управления кадров
      • Противодействие коррупции
      • Защита персональных данных
      • Международное сотрудничество
      • Центр развития профессиональной карьеры
      • СМИ об университете
      • Полезные ссылки
    • Абитуриентам
      • Общая информация
      • Приемная кампания 2023
        • Бакалавриат
        • Специалитет
        • Магистратура
        • Аспирантура
        • Среднее профессиональное образование
      • Информация для иностранных абитуриентов/ Information for foreign applicants
      • Вступительные испытания для инвалидов и лиц с ограниченными возможностями здоровья
      • Стоимость обучения
      • Целевое обучение
      • Образцы заявлений
      • Дни открытых дверей
      • Часто задаваемые вопросы
      • Приемная кампания 2022
        • Бакалавриат
        • Специалитет
        • Магистратура
        • Аспирантура
        • Среднее профессиональное образование
      • Информация о приеме 2021
        • Бакалавриат
        • Специалитет
        • Магистратура
        • Аспирантура
        • Средее профессиональное образование
      • Ответы на обращения абитуриентов
    • Обучение
      • Оценка качества образования
      • Факультеты
      • Кафедры
      • Среднее профессиональное образование
      • Библиотека
        • История библиотеки
        • Правила пользования библиотекой
        • Доступ к электронным образовательным ресурсам и базам данных
        • Электронная библиотека университета
        • Периодические издания
        • Вестник Донского ГАУ
        • Порядок проверки ВКР на объем заимствования и их размещения в ЭБС
        • Информация для пользователей
      • Электронная информационно-образовательная среда
        • Образовательные программы
        • Электронные образовательные ресурсы, базы данных
        • Портфолио студентов
        • Портфолио аспирантов
      • Расписание занятий
      • Аспирантура
        • Документы, регламентирующие образовательный процесс
        • Научное руководство аспирантами и соискателями
        • Образовательные программы
        • Федеральные государственные образовательные стандарты и требования
        • Портфолио аспирантов
        • Контактная информация
      • Платформа дистанционного обучения
      • Обучение иностранных граждан/for foreign students
      • Дополнительное профессиональное образование
      • Заочное обучение
      • Музеи
      • Ответы на вопросы участников образовательного процесса
    • Наука и инновации
      • Национальный проект «Наука и университеты»
      • Инновационные разработки университета
      • Научно-исследовательская база
      • Докторантура
      • Совет молодых ученых и специалистов
        • Состав президиума
        • Документы
        • Новости студенческой науки
      • Защита диссертаций
      • Диссертационный совет 35. 2.014.01
      • Диссертационный совет Д 99.2.099.02
      • Диссертационный совет Д 99.0.104.02
      • Конференции, выставки, семинары, публикации
      • Научные конференции Донского ГАУ
      • Агропромышленный инновационно-консультационный комплекс
      • Вестник университета
      • Гуманитарный Вестник
    • Студенческая жизнь
      • Патриотическое воспитание, противодействие экстремизму и идеологии терроризма
      • Молодежные объединения университета
      • Студенческое самоуправление
      • Студенческий медиа-центр
      • Физкультура и спорт
      • Отдых и творчество
      • Общежития
      • Стипендиальное обеспечение и другие формы материальной поддержки
      • Плата за обучение
      • Социально-психологическая служба
      • Творческое объединение «Донской Пегас»
    • Контакты
    • Электронная информационно-образовательная среда

    Счет 60 «Расчеты с поставщиками и подрядчиками»

    Бухгалтерский счет 60 «Расчеты с поставщиками и подрядчиками» предназначается для формирования данных о произведенных предприятием расчетах с контрагентами по приобретенным ценностям, услугам, выполненным работам. Благодаря системе двойной записи и корреспонденции счета 60, здесь отражаются сведения по акцептованным поставкам и операциям, документы на которые еще отсутствуют (при неотфактурованных сделках). Рассмотрим, в каком порядке ведется учет оприходования имущества или услуг/работ – проводки по сч. 60 вы найдете ниже.

    Счет 60 в бухгалтерском учете

    Основным регламентирующим документом с перечнем счетов организаций является План счетов по приказу № 94н от 31.10.00 г. В соответствии с указанным нормативно-правовым актом бухгалтерский счет 60 «Расчеты с поставщиками и подрядчиками» используется в целях свода данных по поставкам за:

    • Приобретенные ТМЦ, услуги, работы, включая коммунальные, связи, а также акцептованные банком.
    • Поступившие в организацию ТМЦ, работы и/или услуги, документы на которые от поставщиков на момент поступления не получены – неотфактурованные сделки.
    • Образовавшиеся во время приемки излишки ТМЦ.
    • Услуги по доставке ТМЦ, включая переборы/недоборы тарифа/фрахта.
    • Подрядные работы по договорам в строительстве, в области НИОКР и технологических работ.

    Синтетический 60 счет в бухгалтерии позволяет обобщить информацию по всем закупкам за нужный период, а аналитический учет по организациям, документам и видам поступивших ТМЦ/работ/услуг осуществляется с целью формирования детальной картины по взаиморасчетам и источникам поступления ТМЦ в компанию.

    Счет 60 – активный или пассивный?

    Финансовый анализ счета 60 позволяет увидеть, что это активно-пассивный счет. Таким образом, остаток на начало/конец периода может формироваться как по кредиту, так и по дебету. Дебетовое начальное сальдо показывает совокупный размер долга (обязательств) поставщиков/подрядчиков перед организацией. Кредитовое начальное сальдо означает величину долга организации перед контрагентами. Дебет 60 счета показывает, сколько денег за интересуемый промежуток времени перечислено по договорам поставки/подряда/работ, включая суммы предоплаты. Соответственно, кредит 60 счета показывает, на какой объем приобретено ТМЦ, работ или услуг. Аналогичным образом формируются и конечные остатки.

    Обратите внимание! При отражении сведений на сч. 60 «Расчеты с поставщиками» применяется метод начисления, то есть все хозяйственные операции проводятся независимо от факта оплаты – по поступлению ценностей, работ или услуг.

    60 счет – субсчета

    • Счет 60.01 – предназначен для формирования расчетов с контрагентами (поставщиками и/или подрядчиками).
    • Счет 60.02 – служит для формирования данных по перечисленным авансам.
    • Счет 60.03 – используется для отражения сведений по векселям.
    • Счета 60.21, 60.31 – аналоги счета 60.01 для расчетов в валюте/у.е.
    • Счета 60.22, 60.32 – аналоги счета 60.02 для расчетов в валюте/у.е.

    Корреспонденция счета 60

    Бухгалтерская характеристика счета 60 показывает, что такой счет может иметь как кредиторский остаток, так и дебиторский в зависимости от текущего состояния взаиморасчетов. Исходя из сказанного, кредит счета 60 увеличивается при принятии организацией на учет материалов, сырья, топлива, основных средств, товаров, оборудования, а также различных услуг и работ. Корреспонденция выполняется (согласно документации от контрагентов) со счетами учета объектов.

    А что отражается по дебету 60 счета? Это суммы исполнения оговоренных договорными условиями обязательств. То есть оплата полученных от поставщиков/подрядчиков счетов, включая перечисленную предоплату. Аналитическая карточка счета 60 позволяет получить точную информацию о состоянии взаиморасчетов, включая просроченные долги.

    Типовые основные проводки по счету 60

    • Д 60 К 50, 51, 52 – перечислена оплату поставщику по договору (из кассы, с расчетного рублевого счета, с валютного счета).
    • Д счет 60.1 К 62 – отражено проведение взаиморасчета.
    • Д 60 К 66, 67 – отражено переоформление задолженности в заем (краткосрочный/долгосрочный).
    • Д 60 К 91 – отнесена на прочие доходы просроченная дебиторка.
    • Д 07, 08, 10, 41 К 60 – оприходованы поступившие в компанию ценности (оборудование, внеоборотные активы, материалы, товары).
    • Д 19 К 60 – выделен в поставке НДС.
    • Д 20, 25, 26 К 60 – отражены выполненные работы, услуги в себестоимости изделий, общепроизводственных (общехозяйственных) расходах.
    • Д 50, 51, 52 К 60 – оприходованы излишне перечисленные средства от поставщика.
    • Д 94 К 60 – отражена недостача во время приемки ТМЦ.

    Обратите внимание! Регулярная инвентаризация счета 60 должна проводиться перед составлением годовых отчетов по Приказу № 34н (п. 27). 

    Специализация: все виды систем налогообложения, бухотчетность, МСФО

    Эксперт в сфере права, бухучета, финансов и налогообложения. Общий стаж профессиональной деятельности с 2007 года. За это время успешно работала на должностях налогового консультанта, заместителя главного бухгалтера, главного бухгалтера, финансового директора. Автор множества публикаций по практическому применению бухгалтерского, налогового и трудового законодательства для различных профессиональных электронных СМИ. С отличием окончила факультет управления и психологии Кубанского государственного университета и Адыгейский государственный университет по специальности «Бухгалтерский учет и аудит».

    ведущих производителей и поставщиков электрооборудования в США и Канаде

    Изображение предоставлено: forden/Shutterstock.com

    Электрическое оборудование и устройства определяются как любая машина, работающая от электричества, но это определение намеренно оставлено расплывчатым, поскольку существует множество примеров таких машин для конкретных рынков. Электрооборудование широко распространено в современном мире и становится еще более актуальным благодаря Интернету вещей, беспроводной связи и программным приложениям. В этой статье будут рассмотрены самые популярные производители и поставщики бытовых электроприборов, а также запчастей для электрооборудования на сайте Thomas.

    Что такое электрооборудование?

    Электрическое оборудование — это общий термин, используемый для определения любой машины, работающей от электричества, и включает:

    • Освещение
    • Электрические бытовые приборы
    • ИТ-оборудование
    • Двигатели, насосы и системы отопления, вентиляции и кондиционирования
    • Устройства с батарейным питанием
    • Переключатели, реле и компоненты

    и многое другое.

    Электрооборудование также конкретно относится к отдельным компонентам электрического распределительного оборудования, продуктов и систем; однако в этой статье будут рассмотрены вопросы, выходящие за рамки этой категории, поскольку многие производители электрооборудования производят более одного типа продукции для различных отраслей промышленности. Тем не менее, эти машины жизненно важны для правильной работы почти любой другой отрасли и необходимы для большинства, если не для всех современных предприятий сегодня.

    Что такое производители и поставщики электрооборудования?

    Производители и поставщики электрооборудования изготавливают и/или распространяют электрооборудование парным клиентам. Компании могут сосредоточиться только на одном рынке (например, освещение, ИТ, бытовая техника и т. д.) или они могут диверсифицироваться на несколько рынков электрооборудования. Производители и поставщики электрооборудования в настоящее время сосредотачиваются на устойчивости, поскольку глобальные электрические сети должны быть обновлены, чтобы принимать новые формы возобновляемой энергии, а также IoT и усовершенствованные коммуникационные платформы. Независимо от конкретного применения, электрическое оборудование сопровождается подробными инструкциями по установке и обслуживанию и поддерживается службами поддержки клиентов, чтобы гарантировать, что машина работает по назначению.

    Лучшие производители и поставщики электробытовых приборов на Thomas

    В приведенной ниже Таблице 1 перечислены наиболее известные производители и поставщики бытовых электроприборов на сайте Thomasnet. com. В этой информации вы найдете подробную информацию о местонахождении каждой компании, год основания, количество сотрудников и краткое описание деятельности. Прочерки указывают, где данные были недоступны.

    Таблица 1: Наиболее известные производители и поставщики электробытовых приборов на Thomas
    Компания Местоположение Год основания Количество сотрудников
    Allbrands.com Батон-Руж, LA 1976 10-49
    Компания Бетти Миллс (BettyMills.com) Сан-Матео, Калифорния 2002 10-49
    Важные документы Сомердейл, Нью-Джерси
    Корпорация Алмо Филадельфия, Пенсильвания 1946 500-499
    Упаковочная продукция Торрингтон, Коннектикут 1937 100-199
    ООО «Роберт Бош» Фармингтон-Хиллз, MI 1886 1000+
    Оборудование Ace Ок-Брук, Иллинойс 1924 1000+
    Center de Pieces et Services D. B. Инк Монреаль, QC
    Продажа со склада Шарон, Пенсильвания
    Компания Дженерал Электрик Бостон, Массачусетс 1892 10-49

    Наиболее известные производители и поставщики электробытовых приборов на сайте Thomas — резюме компании

    Allbrands.com  продает электрические кухонные приборы, такие как сушилки, соковыжималки, морозильные камеры, стиральные машины, блендеры, миксеры, куттеры и многое другое. Он также предлагает чистящее оборудование и вышивальные машины. Штаб-квартира компании находится в Батон-Руж, штат Луизиана, и работает с 1976 года.

    Компания Betty Mills (BettyMills.com)  продает различные бытовые приборы, включая тостеры, кофеварки, кофеварки, микроволновые печи, холодильники и многое другое. Штаб-квартира компании находится в Сан-Матео, Калифорния, и работает с 2002 года.

    Important Papers   – производитель и дистрибьютор рекламной и рекламной продукции, включая подогреватели кружек, фритюрницы, пылесосы, цифровые термометры для барбекю, пищевые весы и многое другое. Он также предлагает различные услуги, включая проектирование, трафаретную печать, консультации, выполнение и многое другое. Компания базируется в Сомердейле, штат Нью-Джерси.

    Корпорация Almo  занимается продажей бытовой электроники, бытовой техники, профессионального аудио- и видеооборудования и многого другого. Ассортимент продукции включает стиральные машины, плиты, холодильники, телевизоры, микроволновые печи и многое другое. Штаб-квартира компании находится в Филадельфии, штат Пенсильвания, и работает с 19.46. ​​

    Waring Products   – производственная компания, выпускающая портативные коммерческие и бытовые электроприборы, такие как миксеры для напитков, блендеры, оборудование для хранения пищевых продуктов, соковыжималки, кофемолки, вафельницы, тостеры, духовки, горелки и многое другое. Штаб-квартира компании находится в Торрингтоне, штат Коннектикут, и была основана в 1937 году.

    Robert Bosch LLC производит продукты и расходные материалы для мобильных и промышленных технологий, включая системы отопления, оборудование для мобильных устройств, системы для электровелосипедов, бытовую технику и многое другое. Штаб-квартира компании находится в Фармингтон-Хиллз, штат Мичиган, и ведет свою деятельность с 1886 года.0007

    Ace Hardware  распространяет различные инструменты и оборудование, включая электроинструменты, сварочные инструменты, электрическое оборудование, оборудование для уборки, клеи, ленты и многое другое. Штаб-квартира компании находится в Оук-Брук, штат Иллинойс, и была основана в 1924 году.

    Centre de Pieces et Services D.B. Inc.   – дистрибьюторская и сервисная компания, которая поставляет бытовую технику, запасные части, аксессуары и чистящие средства, такие как пылесосы, кухонные приборы, холодильники, микроволновые печи, кухонные приборы, электроинструменты, оборудование для ухода и многое другое. Штаб-квартира компании находится в Монреале, Квебек.

    Складские продажи занимается продажей различного оборудования и расходных материалов, включая бытовую технику, автомобильные запчасти, электротовары, оборудование для систем отопления, вентиляции и кондиционирования воздуха, товары для сада и газона и многое другое. Штаб-квартира компании находится в Шароне, штат Пенсильвания.

    General Electric — крупный конгломерат, который имеет дочерние компании в авиационной, финансовой, цифровой, исследовательской, энергетической, возобновляемой энергетике и других отраслях, и является 33-й -й крупнейшей фирмой в Америке. Производя почти все, что связано с электричеством, GE легко занимает третье место в этом списке как один из крупнейших производителей электрооборудования в мире. Штаб-квартира GE находится в Бостоне, Массачусетс, но она представлена ​​более чем в 170 странах по всему миру.

    Лучшие производители и поставщики запчастей для электрооборудования на Thomas

    В приведенной ниже Таблице 2 перечислены наиболее известные производители и поставщики деталей электрооборудования на сайте Thomasnet. com. В этой информации вы найдете подробную информацию о местонахождении каждой компании, год основания, количество сотрудников и краткое описание деятельности.

    Таблица 2: Наиболее популярные производители и поставщики запчастей для электрооборудования на Thomas
    Компания Местоположение Год основания Количество сотрудников
    Компания Johnson Bros. Roll Forming Co. Беркли, Иллинойс 1948 10-49
    A & S Mold & Die Corp. Чатсуорт, Калифорния 1969 100-199
    Корпорация промышленного вакуумного оборудования Иксония, Висконсин 1992 10-49
    Висконсин Овен Корп. Восточная Троя, Висконсин 1973 100-199
    MME group inc. Vadnais Heights, MN 1974 100-199
    Аркада Штамповка металла Бриджпорт, Коннектикут 1942 50-99
    Американская промышленная компания Герни, Иллинойс 1962 10-49
    ООО «МКМ Композитиз» Манитовок, Висконсин 1983 100-199
    Stangenes Industries, Inc. Пало-Альто, Калифорния 1974 100-199
    Корпорация WAGO Джермантаун, Висконсин 1979 200-499

    Лучшие производители и поставщики запчастей для электрооборудования на сайте Thomas — сводки компаний

    Компания Johnson Bros. Roll Forming Co.  производит как стандартное, так и нестандартное электрическое оборудование, детали и компоненты, изготовленные методом профилирования, включая стержни, кронштейны, панели, уголки, прототипы и многое другое. Штаб-квартира компании находится в Беркли, штат Иллинойс, и работает с 1948 года.

    A&S Mold & Die Corp.  производит электрические и электронные компоненты и детали на заказ. Его спектр услуг включает в себя литье под давлением, цельное литье, высокопрессовое формование, проектирование оснастки, инжиниринг, решения «под ключ», сварку и многое другое. Штаб-квартира компании находится в Чатсуорте, Калифорния, и была основана в 1969.

    Industrial Vacuum Equipment Corp. — производственная компания, которая обеспечивает замену компонентов вакуумного оборудования и электрических принадлежностей. Он также предлагает шнуры, блоки, фонари, жгуты проводов, вилки, держатели предохранителей, элементы управления и многое другое. Штаб-квартира компании находится в Иксонии, штат Висконсин, и работает с 1992 года.

    Wisconsin Oven Corp. является производителем и дистрибьютором, который поставляет электрические компоненты и детали, включая фильтры для вентиляторов, наборы шнуров, приводы переменного тока, ограждения, пускатели двигателей, блоки, валы, автоматические выключатели, контрольные лампы и многое другое. Компания также предлагает услуги по установке и обслуживанию запасных частей. Компания расположена в Восточной Трое, штат Висконсин, и была основана в 1973.

    MME Group Inc. — это компания-производитель на заказ, которая предоставляет контрактные услуги по изготовлению пластиковых деталей и компонентов для электрического и электронного оборудования. Он также предлагает инструментальные, инженерные, сборочные и формовочные услуги. Штаб-квартира компании находится в Ваднайс-Хайтс, штат Миннесота, и была основана в 1974 году.

    Arcade Metal Stamping  — это специализированный производитель и сервисная компания, которая поставляет детали и расходные материалы для электрооборудования, включая балочные зажимы, подвески, опоры коробок, перегородки, удлинительные кольца и многое другое. Он также предлагает различные услуги, такие как проволочная электроэрозионная обработка, клепка, сверление, обработка с ЧПУ и многое другое. Компания базируется в Бриджпорте, штат Коннектикут, и была основана в 1942.

    American Industrial Co.  производит на заказ OEM-детали для электрического и автомобильного оборудования и предлагает такие услуги, как штамповка, сборка, лазерная резка, оснастка, покраска, гальванопокрытие, шлифовка, сверление и многое другое. Штаб-квартира компании находится в Герни, штат Иллинойс, и работает с 1962 года.

    MCM Composites, LLC производит изготовленные методом литья под давлением детали и компоненты для электронного сектора. Он также предлагает услуги по снятию заусенцев, зачистке заусенцев и струйной струйной очистке. Штаб-квартира компании находится в Манитовоке, штат Висконсин, и работает с 19 лет.83.

    Stangenes Industries, Inc.  производит электрические компоненты на заказ, такие как трансформаторы тока и разделительные трансформаторы, источники питания, зарядные катушки индуктивности, делители напряжения и многое другое. Штаб-квартира компании находится в Пало-Альто, Калифорния, и она обслуживает исследовательскую, медицинскую и военную промышленность. Основана в 1974 году.

    WAGO Corp. — производственная фирма, поставляющая различные электронные компоненты, включая реле, переключатели, отвертки, вилки, автоматические выключатели, билеты и многое другое. Штаб-квартира компании находится в Джермантауне, штат Висконсин, и она обслуживает, среди прочего, энергетическую, железнодорожную, автомобильную и морскую отрасли. Он был основан в 1979.

    Ведущие производители и поставщики электрооборудования в США и Канаде — сводка

    В этой статье собраны самые популярные производители и поставщики бытовых электроприборов, а также запчастей для электрооборудования на Томасе. Мы надеемся, что эта информация была вам полезна при поиске поставщика. Чтобы узнать больше об этих компаниях или составить собственный список поставщиков, не стесняйтесь посетить сайт Thomas Supplier Discovery, где у нас есть более 330 поставщиков запчастей для электрооборудования.

    Источники

    1. Ведущие производители электрооборудования: США 2018 | Статистика
    2. Отчет о мировом рынке производства электрооборудования за 2019 г. — ResearchAndMarkets.com | Деловой провод
    3. Крупнейшие в мире компании по производству электрооборудования (power-technology.com)
    4. Электрические и оптоволоконные разъемы Mil-Spec

    Товары других ведущих поставщиков

    • Ведущие производители и поставщики конденсаторов (США и другие страны)
    • Ведущие производители и поставщики трансформаторов в США и во всем мире
    • Лучшие производители и поставщики клапанов в США
    • Ведущие производители медицинского оборудования в США
    • Ведущие производители и поставщики реле в США и во всем мире
    • Ведущие производственные компании США
    • Ведущие производители и поставщики интегральных схем в США и за рубежом
    • Ведущие сервисные компании с ЧПУ в США
    • Ведущие компании-разработчики программного обеспечения машинного зрения в США и мире
    • Ведущие американские и международные производители и компании RFID-систем
    • Лучшие поставщики и производители датчиков движения в США и за рубежом
    • Ведущие компании-разработчики программного обеспечения для анализа методом конечных элементов (FEA) в США и за рубежом
    • Лучшие поставщики и производители автоматических выключателей в США и во всем мире
    • ведущих компаний по автоматизации процессов в США и мире
    • Ведущие производители и поставщики вольфрама и карбида вольфрама в США
    • Ведущие производители и поставщики коаксиального кабеля в США
    • Ведущие биосенсорные компании в США и за рубежом
    • Ведущие компании, предоставляющие услуги 3D-печати в США
    • Ведущие производители антенн RFID в США и за рубежом
    • Ведущие производители термопар в США — по доходам, местонахождению и типу продукции

    Прочие электротехнические изделия

    • Ведущие производители и поставщики трансформаторов в США и во всем мире
    • Ведущие производители и поставщики реле в США и во всем мире
    • Электрические опорные изоляторы
    • Лучшие поставщики и производители автоматических выключателей в США и во всем мире
    • Огнестойкие электрические шкафы
    • Основные электрические разъемы
    • Типы помех в электроснабжении
    • Электрические двухслойные конденсаторы (суперконденсаторы)
    • Как работают сердечники трансформатора
    • Жизнь вне сети — альтернативные источники энергии
    • Типы систем бесперебойного питания (ИБП)
    • Типы электрических выключателей
    • Что такое гидроэлектроэнергия? Взгляд на гидроэнергетический процесс
    • Для чего используется конденсатор? Переменные типы/функции конденсаторов
    • Базовый электрический выключатель, определение, использование и применение
    • Распределительные коробки
    • Типы разъемов электропитания
    • Общие типы трансформаторов
    • Распространенные типы электрических разъемов
    • Все о понижающих повышающих трансформаторах
    • Ведущие производители и поставщики электродвигателей в США
    • Лучший мультиметр
    • Лучшие книги по электротехнике

    Ведущие производители и компании по производству баллонов с пропаном в СШАСледующая история »

    Электрические материалы и изделия | Электротовары оптом

    Электрические материалы или электроприборы — это важные части или элементы, используемые в строительном проекте для подключения вашего дома, офиса или здания к источнику электроэнергии. Электрические части могут варьироваться от небольшой домашней цепи до большой промышленной установки. D&F Liquidators поставляет большой ассортимент электротехнических изделий и электрооборудования, в том числе различные электропроводные материалы и электроматериалы:

  • Электрический кабелепровод и фитинг кабелепровода
  • Электрический кабелепровод представляет собой трубку, по которой проходит электрический провод для питания или связи. Трубопровод изготовлен из жесткой стали; более легкая сталь, называемая IMC или промежуточным трубопроводом; EMT или тонкостенный трубопровод; ПВХ или пластик; алюминий; или с покрытием из ПВХ; который жестко покрыт оболочкой из ПВХ толщиной 40 мил. Кабелепровод для электрических диапазонов размеров от 3/8″ до 6″.

  • Электрические провода и кабели
  • Электрические провода — это металлические, медные или алюминиевые провода, которые передают электричество по электрической цепи. Провод может быть проложен над головой, под землей, через кабелепровод, гибкий или открытый. Провод защищен конструкцией с оболочкой в ​​зависимости от использования. Он также может быть гибким, как при использовании удлинителей. Он может быть очень маленьким, от 26 калибра до 2000 MCM.

  • Взрывозащищенные корпуса
  • Взрывозащищенные — это изделия, в конструкции которых предусмотрено короткое замыкание во избежание воспламенения опасной атмосферы, вызывающей взрыв. Нефтеперерабатывающие заводы, автозаправочные станции, покрасочные камеры — вот примеры того, где могут использоваться эти продукты. По конструкции они изготовлены из сверхпрочной стали или алюминия, а также могут быть изготовлены из стекловолокна.

  • Автоматические выключатели
  • Автоматические выключатели — это устройства, которые автоматически прерывают подачу электрического тока для защиты от короткого замыкания или перегрузки. Автоматические выключатели производятся несколькими производителями, включая Cutler Hammer/ Westinghouse, General Electric, Square D, ITE Sieonans и Federal Pacific. Автоматические выключатели защищают самую маленькую электрическую цепь дома в соответствии с самыми большими промышленными требованиями.

  • Электрические соединители
  • Электрические соединители — это деталь, которая соединяет или адаптирует одну часть к другой. Диапазон размеров разъемов от 3/8″ до 6″. Они могут быть внутренними, наружными, защищенными от коррозии или взрывозащищенными.

  • Электрическая коробка
  • Электрическая коробка представляет собой корпус, используемый для многих целей, таких как протягивание, соединение или завершение электрической цепи. Электрическая коробка может быть установлена ​​винтовой или внутренней; это может быть дождь плотно, или на открытом воздухе. Они могут быть изготовлены из стали, алюминия, пластика, нержавеющей стали или чугуна. Требование может включать защиту от коррозии или взрывозащиту по конструкции.

  • Наконечники
  • Наконечники представляют собой электрические соединители, замыкающие электрическую цепь.

    Критерий вальда пример решения: Критерий Вальда

    Критерий Вальда

    • 1
    • 2
    • 3
    • 4
    • 5
    • 6
    • 7
    • 8
    • 9
    • 10
    • 11
    • 12
    • 13

    Глава 2. Принятие решений в условиях неопределенности

    Критерий Вальда является самым «осторожным». Согласно ему, оптимальной альтернативой будет та, которая обеспечивает наилучший исход среди всех возможных альтернатив при самом плохом стечении обстоятельств.

    Если исходы отражают подлежащие минимизации показатели (убытки, расходы, потери и т.д.), то критерий Вальда ориентируется на «минимакс» (минимум среди максимальных значений потерь всех альтернатив).

    Если в качестве исходов альтернатив фигурируют показатели прибыли, дохода и других показателей, которые надо максимизировать (по принципу «чем больше, тем лучше»), то ищется «максимин» выигрыша (максимум среди минимальных выигрышей). Здесь и далее для всех критериев в тексте мы будем рассматривать именно такой случай, когда исход показывает некий выигрыш.

    По критерию Вальда оценкой i-й альтернативы является ее наименьший выигрыш:

    Wi = min(xij), j = 1..M

    Оптимальной признается альтернатива с максимальным наихудшим выигрышем:

    Х* = Хk, Wk = max(Wi), i = 1..N

    Пример применения критерия Вальда

    Есть два проекта Х1 и Х2, которые при трех возможных сценариях развития региона (j=1..3) обеспечивают разную прибыль. Значения прибыли приведены в таблице 2.2. Необходимо выбрать проект для реализации.

    Табл.2.2. Исходные данные.
    Альтер­нативы (Xi) Состояния природы (j)
    1 2 3
    Х1 45 25 50
    X2 20 60 25

    Среди возможных проектов нет доминирующих ни абсолютно, ни по состояниям. Поэтому решение придется принимать по критериям.

    Если выбор оптимального проекта осуществляется по критерию Вальда, то ЛПР должен выполнить следующие действия:

    1. Найти минимальные исходы для каждой альтернативы. Это и будут значения критерия Вальда:

    W1 = min( x1j ), j = 1..3 => W1 = min(45, 25, 50) = 25

    W2 = min( x2j ), j = 1..3 => W2 = min(20, 60, 25) = 20

    2. Сравнить значения критерия Вальда и найти наибольшую величину. Альтернатива с максимальным значением критерия будет считаться оптимальной:

    25 > 20 => W1 > W2 => X* = X1

    Если бы решение принималось только по критерию Вальда, ЛПР выбрал для реализации проект Х1, поскольку прибыль, которую обеспечит данный проект при самом плохом развитии ситуации, выше.

    Выбрав оптимальную альтернативу по критерию Вальда, ЛПР гарантирует себе, что при самом плохом стечении обстоятельств он не получит меньше, чем значение критерия. Поэтому данный показатель еще называют критерием гарантированного результата.

    Основной проблемой критерия Вальда является его излишняя пессимистичность, и, как следствие, не всегда логичный результат. Так, например, при выборе по данному критерию между альтернативами А{100; 500} и В{90; 1000} следует остановиться на варианте А. Однако в жизни логичнее было бы выбрать В, так как в худшем случае В лишь немного хуже А, тогда как при хорошем стечении обстоятельств В обеспечивает гораздо больший выигрыш.

    Наверх

    • 1
    • 2
    • 3
    • 4
    • 5
    • 6
    • 7
    • 8
    • 9
    • 10
    • 11
    • 12
    • 13

    Дата обновления: 25.09.2014

    Примеры решения задач — Математическое программирование — Исследование операций — ЭММ

    Условие задачи

    Фермер может посеять на данном участке одну из трех культур . Урожайность каждой из культур во многом зависит от погоды, которая может быть засушливой, нормальной или дождливой (влияние других факторов не учитывается). Известна цена  одного центнера культуры , а также урожайности (ц/га) каждой культуры    – урожайность при засушливой погоде,  –урожайность при нормальной погоде,  – урожайность при дождливой погоде. Многолетние наблюдения за погодой данного района показывают, что вероятности засушливой, нормальной и дождливой погоды составляют соответственно .

    Требуется:

    • придать описанной ситуации игровую схему и составить платежную матрицу;
    • пользуясь критериями Бейеса, Вальда, Сэвиджа и Гурвица (величина параметра  для критерия Гурвица задается) выяснить, какую культура следует сеять, чтобы обеспечить наибольший доход.

    Решение задачи

    Игровая схема

    Игра парная,  статистическая. В игре участвуют два игрока. Первый игрок — фермер. Его возможные стратегии:

      — посадить 1-ю культуру

      — посадить 2-ю культуру

      — посадить 3-ю культуру

    Второй игрок — природа.  Под природой  мы  понимаем  совокупность внешних условий, определяющих урожайность. Стратегии природы будут следующими:

     – погода будет засушливая

     – погода будет нормальная

     – погода будет дождливая

     

    Платежная матрица

    Составляем платежную матрицу. Элементы этой матрицы – цена реализации выращенного урожая.

     

    Или

     
    6 9 3
    5 10 30
    8 12 4

     

    Критерий Байеса

    При известных вероятностях воспользуемся критерием Байеса.

    Определяем средние выигрыши:

    Оптимальной  является стратегия 

     

    Критерий Вальда

    Оптимальной является стратегия .

     

    Критерий Сэвиджа

    Составляем матрицу рисков:

           
    2 3 27 27
    3 2 0 3
    0 0 26 26

    Оптимальной является стратегия

     

    Критерий Гурвица

    Согласно критерию Гурвица, наи­лучшим решением является чистая стратегия, соответствующая условию:

    Оптимальной является стратегия

    В соответствии со всеми критериями фермеру необходимо выращивать 2-ю культуру.

    На сайте можно заказать решение контрольной или самостоятельной работы, домашнего задания, отдельных задач. Для этого вам нужно только связаться со мной:

    ВКонтакте
    WhatsApp
    Telegram

    Мгновенная связь в любое время и на любом этапе заказа. Общение без посредников. Удобная оплата переводом на карту СберБанка.

    Подробное решение в электронном виде (docx, pdf) получите точно в срок или раньше.

    Обобщенная оценка и тесты Вальда

    На этой странице

    АннотацияВведениеПриложениеСсылкиАвторское правоСтатьи по теме

    Обобщенная оценка и тесты Вальда описаны и связаны с их необобщенными версиями. Обсуждаются два интересных приложения. В первом выводится новый тест для задачи Беренса-Фишера. Во-вторых, проверяется однородность отклонений от нескольких одномерных нормальных популяций.

    1. Введение

    Этот документ предназначен в качестве учебного пособия для тех, кто хочет получить информацию об обобщенном балле и тестах Вальда. Он расширяет содержание [1] и имеет аналогичные цели; то есть он фокусируется на использовании этих тестов, а не на их свойствах. Он предназначен быть очень доступным. Читателям нужны лишь некоторые предварительные знания о секционированных матрицах, оценках и критериях Вальда, см., например, [1] и [2, глава 3].

    Тест на баллы особенно ценен, когда оценка максимального правдоподобия (ML) в полной модели не предпочтительна, а оценка ML в нулевой модели предпочтительнее. Обратное верно для теста Вальда. Таким образом, когда оценка ML по одной из нулевой и полной моделей не является предпочтительной, тест отношения правдоподобия проблематичен, а один из критериев оценки и теста Вальда — нет. Здесь под нежеланием имеется в виду, что, например, оценки могут быть рассчитаны по какой-либо итерационной схеме с сомнительной сходимостью. Другие возможности заключаются в том, что оценки могут иметь особенно запутанное выражение или свойства конечной выборки (например, большое смещение) могут быть неподходящими для интересующей проблемы.

    Когда оценка машинного обучения как в нулевой, так и в полной моделях нежелательна, нам нужен другой путь вперед. Это обеспечивается обобщенной оценкой и тестами Вальда. Эти тесты особенно ценны, когда модель может быть неправильно определена, но здесь мы не об этом.

    В Разделе 2 описаны обобщенная оценка и тесты Вальда. В разделе 3 этот материал применяется для получения нового теста для задачи Беренса-Фишера, а в разделе 4 рассматривается проверка равенства дисперсий из нескольких независимых нормальных выборок.

    2. М-оценщики и обобщенные оценочные тесты

    Класс М-оценщиков включает как МО, так и метод моментных оценок. M-оценка удовлетворяет

    в котором независимы, но не обязательно одинаково распределены, является известной функцией, не зависящей от или , является -мерным параметром и вообще обозначает вектор нулей. Оценочная функция должна быть достаточно «гладкой». В частности, должны существовать ее производные до второго порядка и их математические ожидания. Следовательно, матрицы и определенные далее предполагаются существующими. Кроме того, ожидание производных второго порядка должно быть ограничено по вероятности. Дополнительные технические подробности о М-оценках можно найти в [3, глава 5].

    В нашей настройке мы предполагаем, что и что мы хотим проверить : против альтернативы : с вектором основного интереса, с вектором мешающих параметров и с . Обобщенный балльный тест основан на частичной М-оценке, которая удовлетворяет

    где разбивается аналогично , так что , и где в которой находится М-оценка при нулевой гипотезе. Определять в котором обозначает математическое ожидание при нулевой гипотезе. Вот и есть и есть. Заметим, что не обязательно симметричный while есть. Это означает, что форму обобщенных тестов, приведенную, например, в [4], необходимо немного видоизменить. Обобщенная статистическая оценка теста определяется выражением

    в котором, как легко показать, и аргументы в , и опущены; здесь все такие. Точно так же обобщенная статистика теста Вальда определяется выражением

    в котором все аргументы . В изложении [4] параметры оцениваются ML, но данные не поступают из параметрической модели: это ML при неправильной спецификации. В [5] даны определения Кента, но вместо ML-оценок разрешены любые M-оценки. В [4] также отмечается, что и на практике могут быть заменены любыми непротиворечивыми оценками.

    Альтернативный, более удобный для расчета вид приведен в [2], где он применяется для построения обобщенных гладких критериев согласия. Эта форма дает

    в котором Эквивалентность двух форм требует рутинной, но утомительной матричной алгебры и здесь не рассматривается. Асимптотическое распределение обоих и ниже равно .

    Если — производная логарифма вероятности, которая является обычной функцией оценки, то — обычная (симметричная) информационная матрица, и . Если используется оценка ML, то обычная статистика критерия Вальда и обычная статистика критерия оценки. Оба приведены в таком виде в [1]. Подробнее см. [5, 6].

    В [5, стр. 328] рекомендуется заменить обратную асимптотическую ковариационную матрицу на обобщенную обратную непротиворечивой оценки. Хотя это может показаться тривиальным, при вычислении любого обычного или обобщенного показателя или статистики критерия Вальда мы находим где по крайней мере асимптотически многомерная норма и по крайней мере асимптотически ковариационная матрица полного ранга . Очень иногда бывает удобнее найти точную ковариационную матрицу, а не асимптотически эквивалентную. Если это так, то в приведенных выше выражениях можно использовать точную ковариационную матрицу; аналогичным образом, когда это уместно, может использоваться обобщенная обратная точная или асимптотически эквивалентная ковариационная матрица.

    3. Проблема Беренса-Фишера

    В задаче Беренса-Фишера — случайная выборка из совокупности и независимая случайная выборка из совокупности. Желательно протестировать : против : , со стандартными отклонениями и мешающими параметрами. В [2, Пример  3.3.2] получены отношение правдоподобия, оценка и критерии Вальда. Тест на оценку требует решения неудобного кубического уравнения; так что это одна из ситуаций, в которой статистика Вальда выглядит явно более привлекательной, чем как отношение правдоподобия, так и статистика тестов.

    Когда оценочной функцией является обычная функция оценки, обобщенный тест оценки является обычным тестом оценки. Чтобы соответствовать нашим обозначениям положить , , и . Мы тестируем против , с неприятными параметрами и . Логарифм вероятности равен

    . и, следовательно, функция оценки имеет следующие компоненты: Это частные производные логарифма вероятности. При нулевой гипотезе оценочные уравнения имеют вид . Это приводит к неудобному кубическому уравнению, упомянутому ранее. Если мы продолжим эту модель, кубическое должно быть решено, чтобы найти , и, следовательно, и . Мы также находимоткуда

    а обобщенная статистика тестовой оценки равна . Это просто обычная статистика тестов.

    Хотя решение кубика не представляет большой трудности, если мы изменим его так, чтобы он стал можно найти, возможно, менее эффективную, но определенно более удобную оценку общего среднего при нулевой гипотезе. Эта оценка является решением , а именно . Если мы также изменим так, чтобы оставляя два других уравнения без изменений, тест на обобщенную оценку основан на Оценки и немного отличаются от тех, которые были найдены ранее, будучи и . Изменение предыдущего вывода дает откуда

    а статистика обобщенного теста равна

    . Можно показать, что статистика теста Вальда является функцией один-один от этого , так что эти два теста эквивалентны. Однако, если использовать асимптотические критические значения, тест с обобщенной оценкой имеет фактические размеры тестов, намного более близкие к номинальным размерам, чем тест Вальда. При использовании смоделированных критических значений, которые практически точны, мощность теста с обобщенной оценкой находится в пределах 1% от укоренившегося теста из-за Уэлча [7]. Таким образом, по этому критерию тест Уэлча и обобщенный балльный тест практически неотличимы.

    Тест Уэлча очень похож на тест Вальда. Использование приближения Саттертуэйта к нулевому распределению критерия Уэлча дает превосходное соответствие между номинальным и фактическим размерами теста. Однако приближение Саттертуэйта не так хорошо работает для . Следовательно, с точки зрения соответствия между номинальным и фактическим тестовыми размерами с использованием аппроксимаций и асимптотических критических значений предпочтение следует отдавать критерию Уэлча. Поддержка этих утверждений и дополнительные численные детали доступны в [8].

    4. Проверка равенства дисперсий

    Предположим, что у нас есть независимые случайные выборки с th, , размером и из нормальной совокупности. Общий размер выборки составляет . Мы стремимся проверить равенство отклонений: : скажем против альтернативы : не . Популярные тесты включают тест отношения правдоподобия, часто называемый тестом Бартлетта, и тест Левена. Известно, что первый ненадежен, а второй более надежен в том смысле, что его фактические уровни ближе к номинальным уровням. Тест Левена менее эффективен, чем тест Бартлетта, когда данные согласуются с нормой.

    Теперь мы построим тест Вальда против . Мы могли бы использовать построение обобщенного критерия Вальда с производной логарифма вероятности, но мы оставляем это в качестве упражнения для заинтересованного читателя. Мы могли бы также вычислить одну из форм асимптотической ковариационной матрицы, но это тот случай, когда проще вычислить точную ковариационную матрицу. Более того, точная ковариационная матрица включает неудобную обратную; поэтому вместо этого мы используем обратную модель Мура-Пенроуза. Это определено в приложении вместе с некоторыми соответствующими полезными результатами. Этот подход приводит к более простой тестовой статистике.

    В этом примере, поскольку мы вычисляем статистику теста Вальда, все оценки являются ML. Как следствие, оценки обозначаются шляпами () вместо тильд (). Мы также используем несмещенные версии выборочных дисперсий (с делителями вместо ). Они асимптотически эквивалентны обычным оценкам ML, а соответствующая статистика теста асимптотически эквивалентна обычной статистике теста Вальда.

    Прежде чем приступить к построению, обратите внимание, что если является несмещенной выборкой дисперсии случайной выборки размера из распределения, то имеет распределение. Как известно, , так что Из теоремы Рао-Блэквелла оптимальная оценка , будучи единственной оценкой с минимальной дисперсией в классе несмещенных оценок . Эта оптимальность определяется при оценке . Запись для несмещенной оценки дисперсии генеральной совокупности , оптимальная оценка для .

    Если нулевая гипотеза верна, несмещенной оценкой общей дисперсии генеральной совокупности является дисперсия объединенной выборки, где для . Обратите внимание, что поскольку , . Теперь определите Затем . Это ноль тогда и только тогда, когда для всех . Следовательно, проверка на равенство дисперсий эквивалентна проверке : против : . Непредвзятая оценка есть и поскольку симметрична, имеет ковариационную матрицу, оцениваемую где теперь . Теперь CDC не имеет полного ранга, и для использования результатов о квадратичных формах многомерных нормальных случайных величин требуются обобщенные или псевдообратные. Здесь мы используем обратную матрицу Мура-Пенроуза M. См. приложение.

    Поскольку это идемпотент, обратная функция Мура-Пенроуза для CDC задается формулой

    Статистика теста Вальда для тестирования : против : есть Так как , следует сравнивать с распределением для оценки значимости. Если тест показывает значимость на соответствующем уровне, можно провести грубые попарные сравнения, как при сравнении средних в дисперсионном анализе. Чтобы увидеть, как это сделать, сначала обратите внимание, что, как и выше, имеет распределение, которое для больших примерно равно . Отсюда приблизительно и при нулевой гипотезе равенства дисперсий для любого разность приблизительно равна , и может быть оценена как . Наименее значимое различие может быть построено обычным способом.

    Приложение
    Обратное Мура-Пенроуза

    Одной из нескольких псевдообратных или обобщенных обратных является обратная Мура-Пенроуза: см., например, [9, раздел   8.11]. Единственная обратная Мура-Пенроуза вещественной симметричной матрицы B удовлетворяет Привычно показывать следующее.

    (i) Если

    , то

    . (ii) Если H ортогонален, то

    . (iii) Если A идемпотент, то

    . (iv) Если последующие матричные произведения определены, то

    и

    .

    Хорошо известно, что если с рангом , то имеет распределение где является псевдообратным . Для сценария здесь разумно протестировать использование тестовой статистики.

    Ссылки
    1. JCW Rayner, «Асимптотически оптимальные тесты», Journal of the Royal Statistical Society Series D , vol. 46, нет. 3, стр. 337–346, 1997.

      Посмотреть по адресу:

      Сайт издателя | Академия Google

    2. J. C. W. Rayner, O. Thas, and D. J. Best, Smooth Tests of Goodness of Fit: Using R , John Wiley & Sons, Сингапур, 2-е издание, 2009 г.

    3. A. W. van der Vaart, 90symotic Статистика , том. 3 of Cambridge Series in Statistical and Probabilistic Mathematics , Cambridge University Press, Cambridge, UK, 1998.0101 Биометрика , том. 69, нет. 1, стр. 19–27, 1982.

      Посмотреть по адресу:

      Google Scholar | Zentralblatt МАТЕМАТИКА | MathSciNet

    4. Д. Боос, «Об обобщенных тестах оценки», The American Statistician , vol. 47, стр. 327–333, 1992.

      Посмотреть по адресу:

      Google Scholar

    5. Х. Уайт, «Оценка максимального правдоподобия неверно заданных моделей», Econometrica , vol. 50, нет. 1, стр. 1–25, 1982.

      Посмотреть по адресу:

      Сайт издателя | ученый Google | Zentralblatt МАТЕМАТИКА | MathSciNet

    6. Б. Л. Уэлч, «Значение разницы между двумя средними значениями, когда дисперсии совокупности не равны», Biometrika , vol. 29, pp. 350–362, 1937.

      Посмотреть по адресу:

      Google Scholar

    7. П. Риппон, Дж. Рейнер и О. Тас, «Конкурент теста Уэлча в задаче Беренса-Фишера. », Неопубликованный отчет, 2008 г.

      Посмотреть по адресу:

      Google Scholar

    8. Дж. Л. Голдберг, Matrix Theory with Applications , McGraw-Hill, New York, NY, USA, 1991.

    Это статья в открытом доступе, распространяемая под Лицензия Creative Commons Attribution, которая разрешает неограниченное использование, распространение и воспроизведение на любом носителе при условии надлежащего цитирования оригинальной работы.

    12.1 — Логистическая регрессия | СТАТ 462

    Логистическая регрессия моделирует взаимосвязь между предикторными переменными и категориальной переменной отклика. Например, мы могли бы использовать логистическую регрессию для моделирования взаимосвязи между различными измерениями изготовленного образца (такими как размеры и химический состав), чтобы предсказать, произойдет ли трещина более 10 мил (бинарная переменная: либо да, либо нет). Логистическая регрессия помогает нам оценить вероятность попадания в определенный уровень категориального ответа с учетом набора предикторов. Мы можем выбрать один из трех типов логистической регрессии в зависимости от характера категориальной переменной отклика:

    Двоичная логистическая регрессия :

    Используется, когда ответ является бинарным (т. е. имеет два возможных результата). В приведенном выше примере взлома будет использоваться бинарная логистическая регрессия. Другие примеры бинарных ответов могут включать прохождение или непрохождение теста, ответ «да» или «нет» в опросе и наличие высокого или низкого артериального давления.

    Номинальная логистическая регрессия :

    Используется при наличии трех или более категорий без естественного порядка уровней. Примеры номинальных ответов могут включать в себя отделы компании (например, маркетинг, продажи, отдел кадров), тип используемой поисковой системы (например, Google, Yahoo!, MSN) и цвет (черный, красный, синий, оранжевый).

    Порядковая логистическая регрессия :

    Используется при наличии трех или более категорий с естественным порядком уровней, но ранжирование уровней не обязательно означает, что интервалы между ними равны. Примерами порядковых ответов могут быть то, как учащиеся оценивают эффективность курса обучения в колледже (например, «хорошо», «средне», «плохо»), уровень вкуса горячих крылышек и состояние здоровья (например, «хорошее», «стабильное», «серьезное», «критическое»).

    Особые проблемы с моделированием категориальной переменной отклика включают ненормальные условия ошибки, непостоянную дисперсию ошибки и ограничения на функцию отклика (т. е. ответ находится в диапазоне от 0 до 1). Здесь мы исследуем способы решения этих проблем в условиях бинарной логистической регрессии. Номинальная и порядковая логистическая регрессия в этом курсе не рассматриваются.

    Множественная бинарная логистическая регрессионная модель выглядит следующим образом:

    \[\begin{align}\label{logmod}
    \pi(\textbf{X})&=\frac{\exp(\beta_{0 }+\beta_{1}X_{1}+\ldots+\beta_{k}X_{k})}{1+\exp(\beta_{0}+\beta_{1}X_{1}+\ldots+\ beta_ {k} X_ {k})} \ notag \\
    и
    = \ frac {\ exp (\ textbf {X} \ beta)} {1+ \ exp (\ textbf {X} \ beta)} \\
    и
    =\frac{1}{1+\exp(-\textbf{X}\beta)},
    \end{align}\]

    , где здесь \(\pi\) обозначает вероятность, а — нет иррациональное число 3. 14….

    • \(\pi\) — это вероятность того, что наблюдение относится к определенной категории двоичной Y переменной, обычно называемой «вероятностью успеха».
    • Обратите внимание, что модель описывает вероятность события как функцию X переменных. Например, он может предоставить оценки вероятности того, что у пожилого человека есть болезнь сердца.
    • В логистической модели оценки $\pi$ из уравнений, подобных приведенному выше, всегда будут находиться в диапазоне от 0 до 1. Причины:
      • Числитель \(\exp(\beta_{0}+\beta_{1}X_{1}+\ldots+\beta_{k}X_{k})\) должен быть положительным, так как это степень положительное значение ( e ).
      • Знаменатель модели равен (1 + числитель), поэтому ответ всегда будет меньше 1.
    • С одной переменной X теоретическая модель для \(\pi\) имеет вытянутую форму «S» (или сигмоидальную форму) с асимптотами в точках 0 и 1, хотя в выборочных оценках мы можем не увидеть эту форму «S» если диапазон 9{n}[y_{i}\textbf{X}_{i}\beta-\log(1+\exp(\textbf{X}_{i}\beta))].
      \end{align*}\]

      Максимизация правдоподобия (или логарифмического правдоподобия) не имеет решения в закрытой форме, поэтому для нахождения оценки коэффициентов регрессии $\hat{\ бета}$.

      Для иллюстрации рассмотрим данные, опубликованные о n = 27 больных лейкемией. Данные (leukemia_remission.txt) содержат переменную ответа о том, имела ли место ремиссия лейкемии (REMISS), которая определяется числом 1,9.0003

      Прогностические переменные: клеточность среза костного мозга (CELL), дифференциальный процент бластов в мазке (SMEAR), процент абсолютного лейкозно-клеточного инфильтрата (INFIL), процентный индекс мечения клеток лейкоза костного мозга (LI), абсолютный количество бластов в периферической крови (BLAST) и самая высокая температура до начала лечения (TEMP).

      Следующие выходные данные показывают оценочное уравнение логистической регрессии и связанные с ним тесты значимости

      Коэффициенты
      Термин        Coef  SE Coef       95% CI      Z-значение  P-значение     VIF
      Константа    64,3     75,0  (-82,7, 211,2)     0,86     0,391 90 2   52,1  (-71,4, 133,0)     0,59    0,554   62,46
      МАЗОК       24,7     61,5  (-95,9 , 145,3)     0,40    0,688  434,42
      INFIL      -25,0     65,3 (-152,9, 103,0)    -0,38    0,702  471,10
      LI 5,         4,36 9,57)     1,64    0,101    4,43
      ВЗРЫВ      -0,01     2,27  ( -4,45,  4,43)    -0,01    0,996    4,18
      TEMP      -100,2     77,8  (-252,6,  52,2)    -1,29    0,198    3,01

      Критерий Вальда

      используем т -тесты в линейной регрессии). Для оценок максимального правдоподобия отношение })}
      \end{equation*}\]

      можно использовать для проверки $H_{0}: \beta_{i}=0$. Стандартная нормальная кривая используется для определения значения $p$ теста. Кроме того, доверительные интервалы могут быть построены как

      \[\begin{equation*}
      \hat{\beta}_{i}\pm z_{1-\alpha/2}\textrm{s.e.}(\hat{\beta}_{i}).
      \end{equation*}\]

      Оценки коэффициентов регрессии, $\hat{\beta}$, приведены в таблице коэффициентов в столбце с надписью «Coef». В этой таблице также приведены значения коэффициента p , основанные на критериях Вальда. Индекс клеток лейкемии костного мозга (LI) имеет наименьшее значение p и, таким образом, оказывается наиболее близким к достоверному прогностическому признаку наступления ремиссии. Изучив различные подмножества данных, мы обнаруживаем, что хорошей моделью является та, которая включает только индекс маркировки в качестве предиктора:

      Коэффициенты.  ( 0,57,  5,22)     2,44    0,015  1,00

      Уравнение регрессии
      P(1) = exp(Y’)/(1 + exp(Y’))
      Y’ = -3,78 + 2,90 LI

      визуализировать сигмоидальную форму подобранной кривой логистической регрессии:

      Шансы, логарифм шансов и отношение шансов

      Существуют алгебраически эквивалентные способы записи модели логистической регрессии: pi}{1-\pi}=\exp(\beta_{0}+\beta_{1}X_{1}+\ldots+\beta_{k}X_{k}),
      \end{уравнение}\]

      , которое представляет собой уравнение, описывающее шансы попасть в текущую интересующую категорию. По определению, 90 189 шансов 90 190 для события составляют 90 225 π 90 226  / (1 —  π ), так что P — это вероятность события. Например, если вы находитесь на ипподроме и есть вероятность 80%, что определенная лошадь выиграет скачки, то ее шансы равны 0,80 / (1 — 0,80) = 4, или 4:1.

      Второй

      \[\begin{equation}\label{logmod2}
      \log\biggl(\frac{\pi}{1-\pi}\biggr)=\beta_{0}+\beta_{ 1}X_{1}+\ldots+\beta_{k}X_{k},
      \end{equation}\]

      , в котором утверждается, что (натуральный) логарифм шансов является линейной функцией X переменных (и часто называется логарифмическими шансами ). Это также называется логит-преобразованием вероятности успеха, \(\pi\).

      отношение шансов (которое мы запишем как $\theta$) между шансами для двух наборов предикторов (скажем, $\textbf{X}_{(1)}$ и $\textbf{X}_{ (2)}$) задается как

      \[\begin{equation*}
      \theta=\frac{(\pi/(1-\pi))|_{\textbf{X}=\textbf{X }_{(1)}}}{(\pi/(1-\pi))|_{\textbf{X}=\textbf{X}_{(2)}}}.
      \end{equation*}\]

      Для бинарной логистической регрессии шансы на успех составляют:

      \[\begin{equation*}
      \frac{\pi}{1-\pi}=\exp(\ textbf{X}\бета).
      \end{equation*}\]

      Подставив это в формулу для $\theta$ выше и установив $\textbf{X}_{(1)}$ равным $\textbf{X}_{(2 )}$ за исключением одной позиции (т. е. только один предиктор отличается на одну единицу), мы можем определить связь между этим предиктором и ответом. Отношение шансов может быть любым неотрицательным числом. Отношение шансов, равное 1, служит базой для сравнения и указывает на отсутствие связи между ответом и предиктором. Если отношение шансов больше 1, то шансы на успех выше для более высоких уровней непрерывного предиктора (или для указанного уровня фактора). В частности, шансы мультипликативно увеличиваются на $\exp(\beta_{j})$ при каждом увеличении $\textbf{X}_{j}$ на одну единицу. Если отношение шансов меньше 1, то шансы на успех меньше для более высоких уровней непрерывного предиктора (или для указанного уровня фактора). Значения дальше от 1 представляют более сильные степени ассоциации.

      Например, когда имеется только один предиктор \(X\), шансы на успех составляют:

      \[\begin{equation*}
      \frac{\pi}{1-\pi}=\ exp(\beta_0+\beta_1X).
      \end{equation*}\]

      Если мы увеличим \(X\) на одну единицу, отношение шансов составит

      \[\begin{equation*}
      \theta=\frac{\exp(\beta_0+\ beta_1(X+1))}{\exp(\beta_0+\beta_1X)}=\exp(\beta_1).
      \end{equation*}\]

      Чтобы проиллюстрировать это, соответствующие выходные данные примера с лейкемией:

      Отношения шансов для непрерывных предикторов 9{\textrm{th}}$ процентиль от стандартного нормального распределения. Интерпретация отношения шансов заключается в том, что при каждом увеличении LI на 1 единицу расчетные шансы ремиссии лейкемии умножаются на 18,1245. Однако, поскольку LI находится между 0 и 2, может быть, имеет смысл сказать, что на каждые 0,1 единицы увеличения L1 расчетные шансы ремиссии умножаются на $\exp(2,89726\times 0,1)=1,336$. Тогда

      • При LI=0,9 предполагаемый шанс ремиссии лейкемии составляет $\exp\{-3,77714+2,89726*0,9\}=0,310$.
      • При LI=0,8 предполагаемая вероятность ремиссии лейкемии составляет $\exp\{-3,77714+2,89726*0,8\}=0,232$.
      • Полученное отношение шансов составляет $\frac{0,310}{0,232}=1,336$, что представляет собой отношение шансов на ремиссию при LI=0,9 по сравнению с шансами при L1=0,8.

      Обратите внимание, что $1,336\times 0,232=0,310$, что демонстрирует мультипликативный эффект $\exp(0,1\hat{\beta_{1}})$ на шансы.

      Тест отношения правдоподобия (или отклонения)

      тест отношения правдоподобия  используется для проверки нулевой гипотезы о том, что любое подмножество $\beta$ равно 0. Количество $\beta$ в полной модели равно k +1 , в то время как количество $\beta$ в сокращенной модели равно r +1 . (Помните, что сокращенная модель — это модель, которая получается, когда $\beta$ в нулевой гипотезе установлены равными 0. ) Таким образом, количество $\beta$, проверяемых в нулевой гипотезе, равно \((k +1)-(г+1)=к-г\). Тогда статистика теста отношения правдоподобия определяется как: 92$ = отклонение (уменьшенное) – отклонение (полное).

      Эта процедура тестирования аналогична общей процедуре линейного F-теста для множественной линейной регрессии. Однако обратите внимание, что при тестировании одного коэффициента критерий Вальда и критерий отношения правдоподобия будут , а не , в целом дадут идентичные результаты.

      Чтобы проиллюстрировать это, соответствующие выходные данные программного обеспечения для примера с лейкемией:9     8,299        8,30    0,004
      LI          1      8,299     8,299        8,30    0,004
      Ошибка          25            25 2 26,073 3      1,04 34,372

      Поскольку в этом примере имеется только один предиктор, эта таблица просто предоставляет информацию о тесте отношения правдоподобия для LI ( p — значение 0,004), что похоже, но не идентично более раннему результату теста Вальда ( p — значение 0,015). Таблица отклонений включает следующее:

        9009{2}$ распределение с \(2-1=1\) степенями свободы.

      При использовании теста отношения правдоподобия (или отклонения) для более чем одного коэффициента регрессии мы можем сначала подобрать «полную» модель, чтобы найти отклонение (полное), которое показано в строке «Ошибка» в результирующей полной модели. Отклонение Стол. Затем подгоните «урезанную» модель (соответствующую модели, которая получается, если нулевая гипотеза верна), чтобы найти отклонение (уменьшенное), которое показано в строке «Ошибка» в результирующей таблице отклонений сокращенной модели. Например, соответствующие таблицы отклонений для примера вспышки заболевания на страницах 581–582 из 9.0225 Прикладные модели линейной регрессии (4-е изд.) Катнера и др.:

      Полная модель:

      Источник      DF Adj Dev Adj Среднее значение хи-квадрат P-значение 0217 Ошибка       88   93,996   1,06813
      Всего       97
      Регрессия 054    1,0866 92 = 101,054-93,996 = 7,058\), которое сравнивается с распределением хи-квадрат с \(10-5=5\) степенями свободы, чтобы найти p -значение = 0,216 > 0,05 (что означает, что условия взаимодействия равны не имеет значения при уровне значимости 5%). 2\). Например, соответствующая таблица отклонений для примера вспышки заболевания: 9
      Регрессия      7,40    0,007               1    1,804    1,8040        1,80    0,179
       7   Сектор          1   10,448   10,4481       10,45    0,001
      Возраст*Средний      1    4,570    4,5697        4,57    0,033
        Возраст*Младший       1    1,015    1,0152        1,02    0,314 92 = 4,570+1,015+1,120+0,000+0,353 = 7,058\), то же, что и в первом расчете.

        Тесты на соответствие

        Общие характеристики подобранной модели можно измерить с помощью нескольких различных тестов на соответствие. Двумя тестами, требующими повторных данных (множество наблюдений с одинаковыми значениями для всех предикторов), являются критерий согласия хи-квадрат Пирсона и критерий согласия отклонения (аналогичный множественной линейной регрессии). F-критерий несоответствия). Оба этих теста имеют статистику, приблизительно распределенную по принципу хи-квадрат с c k  — 1 степеней свободы, где c  – количество различных комбинаций переменных-предикторов. Когда тест отклоняют, имеет место статистически значимое несоответствие. В противном случае нет никаких признаков несоответствия.

        Напротив, критерий согласия Хосмера-Лемешова полезен для нереплицированных наборов данных или для наборов данных, содержащих лишь несколько повторяющихся наблюдений. Для этого теста наблюдения группируются на основе их предполагаемых вероятностей. Результирующая тестовая статистика приблизительно распределена по хи-квадрат с c  —  2  степеней свободы, где c  – количество групп (обычно выбирается от 5 до 10, в зависимости от размера выборки).

        Чтобы проиллюстрировать это, соответствующие выходные данные программного обеспечения для примера с лейкемией:     23,93    0,523
        Хосмер-Лемешоу   7        6,87    0,442

        Поскольку для этого примера нет реплицированных данных, критерий отклонения и согласия Пирсона недействителен, поэтому первые две строки этой таблицы следует игнорировать. Однако тест Хосмера-Лемешоу не требует дублирования данных, поэтому мы можем интерпретировать его высокие 9{2} = \ frac {\ ell (\ hat {\ beta_ {0}}) — \ ell (\ hat {\ beta})} {\ ell (\ hat {\ beta_ {0}}) — \ ell_ { S}(\beta)},
        \end{equation*}\]

        где $\ell(\hat{\beta_{0}})$ — логарифмическая вероятность модели, когда включен только отрезок и $ \ell_{S}(\beta)$ — логарифмическая вероятность насыщенной модели (т. е. когда модель идеально соответствует данным). Этот R 2 действительно идет от 0 до 1, где 1 идеально подходит. Для нереплицированных данных $\ell_{S}(\beta)=0$, поэтому формула упрощается до: 9{2}=1-\фракция{26,073}{34,372}=0,2414.
        \end{equation*}\]

          Необработанный остаток

          Необработанный остаток представляет собой разницу между фактическим ответом и расчетной вероятностью из модели. Формула для исходного остатка:

          \[\begin{equation*}
          r_{i}=y_{i}-\hat{\pi}_{i}.
          \end{equation*}\]

          Остаток Пирсона

          Остаток Пирсона корректирует неравную дисперсию необработанных остатков путем деления на стандартное отклонение. Формула остатков Пирсона равна 9.0003

          \[\begin{equation*}
          p_{i}=\frac{r_{i}}{\sqrt{\hat{\pi}_{i}(1-\hat{\pi}_{i })}}.
          \end{equation*}\]

          Остатки отклонения

          Остатки отклонения также популярны, потому что сумма квадратов этих остатков является статистикой отклонения. Формула для остатка отклонения:

          \[\begin{equation*}
          d_{i}=\pm\sqrt{2\biggl[y_{i}\log\biggl(\frac{y_{i}}{ \шляпа{\pi}_{i}}\biggr)+(1-y_{i})\log\biggl(\frac{1-y_{i}}{1-\шляпа{\pi}_{i }}\biggr)\biggr]}.
          \end{equation*}\]

          Вот графики остатков Пирсона и остатков отклонения для примера лейкемии. На этих графиках нет тревожных закономерностей, указывающих на серьезную проблему с моделью.

          Шляпные значения

          Шляпная матрица служит той же цели, что и в случае линейной регрессии — для измерения влияния каждого наблюдения на общее соответствие модели — но интерпретация не столь ясна из-за ее более сложная форма. Значения шляпы (рычаги) даны 9{\ textrm {T}} \ textbf {W} \ textbf {X}) \ textbf {x} _ {i},
          \ end {equation *} \]

          , где W — это $ n \ раз n $ диагональная матрица со значениями $\hat{\pi}_{i}(1-\hat{\pi}_{i})$ при $i=1 ,\ldots,n$ на диагонали.

        Калькулятор умножение на дробь: Калькулятор рациональных выражений

        Умножение дробей через калькулятор онлайн

        Дроби – это определенная форма исчисления, которая часто используется для выполнения различных математических операций. Дроби, как правило, используются для умножения, деления, вычитания или прибавления. Дробная черта обычно используется для того, чтобы разделить разные части дроби. Например, верхнюю часть обычно называют числителем, а нижнюю – знаменателем.

        Все дроби могут отличаться своими особенностями и характеристиками. Например, бывают такие дроби, в которых числитель обычно больше знаменателя. В некоторых случаях бывают обратные ситуации. Такие дроби считаются неправильными, поэтому для работы с ними применяются некоторые другие правила, которые позволяют пользователю выполнить определенные операции. Бывают также смешанные дроби, которые представляют собой целое число и дробную часть. С ними можно также выполнять операции умножения по определенным правилам. Для этого нужно внимательно изучить алгоритм, чтобы предпринять соответствующие действия.

        Умножение дробей – это классическая операция, которая используется для того, чтобы умножить одну дробь на другое. Для выполнения поставленной задачи следует знать определенные правила. Если следовать всем рекомендациям, можно легко выполнить все процедуры с минимальными затратами. Также можно воспользоваться специальными онлайн-калькуляторами, которые помогут решить задачу намного быстрее. При этом важно понимать базовые принципы выполнения операции, чтобы вникнуть в механику действий.

        Как умножить дробь на дробь?

        Это классический пример, который часто используется в математике для выполнения соответствующих действий. Чтобы выполнить подобную операцию, достаточно выполнить некоторые базовые рекомендации. Умножить дроби можно следующим образом. Для этого необходимо отдельно выполнить операцию умножения числителей и знаменателей. В результате должно получится новое число, которое будет результатом умножения числителя и знаменателя.

        Часто в результате умножения простых дробей может возникнуть такая ситуация, когда дробь можно сократить. В таком случае значения действительно можно сократить для того, чтобы получить более общее значение. Также можно привести дроби к общему знаменателю, что поможет уменьшить полученное значение, а также упростить дальнейшие математические манипуляции. В качестве примера можно рассмотреть такие дроби:

        7/15 х 10/9 = (7 х 10)/(15 х 9) = 70/135 = 14/27.

        Как видно из описанного выше примера, сначала были выполнены операции отдельного умножения числителя и знаменателя. После этого Полученный результат был сокращен для получения более аккуратного и чистого результата, который можно использовать для выполнения дальнейших математических исчислений любой сложности. Если воспользоваться калькулятором умножения дробей, можно существенно упростить выполнение операции, минимизировав затраты.

        Как умножить смешанные дроби?

        Для выполнения операции со смешанными дробями, необходимо знать определенные правила. Для этого следует обратить внимание на следующие базовые правила:

        • для начала необходимо смешанную дробь преобразовать в неправильную – для этих целей нужно выполнить несколько простых манипуляций;
        • далее необходимо обязательно умножить отдельно числители и знаменатели, как это было выполнено в наиболее простом случае с обычными дробями;
        • при необходимости полученный результат сокращают на определенное значение;
        • если получена неправильная дробь в результате выполнения соответствующих манипуляций, тогда необходимо преобразовать его в смешанный – обычно эта процедура не требует много времени и усилий.

        Если следовать всем перечисленным рекомендациям, получится быстро решить задачу с минимальными усилиями. Для большего понимания особенностей операции следует внимательно изучить пример:

        2 1/2 х 1 2/3 = (2 х 2 + 1) / 2 х (1 х 3 + 2) / 3 = 5/2 х 5/3 = 25/6 = 4 1/6.

        Как видно, операция умножения смешанных дробей выполняется по формуле, поэтому вы можете более подробно рассмотреть данный пример. Можно легко выполнить умножение дробей онлайн с помощью специального калькулятора, что позволит избежать наиболее распространенных ошибок, а также снизить риски получения неправильных результатов, что может оказать неприятные последствия в более сложных задачах.

        Как умножить дробь на число?

        Как правило, умножение дробей на целые числа практически ничем не отличается от обычной процедуры умножения дробей. Для выполнения операции необходимо преобразовать обычное число в дробь, приведя все к общему знаменателю для того, чтобы было проще выполнения операцию умножения. Для лучшего понимания данного примера необходимо обратить внимание на такой простой пример:

        5 х 1 1/2 = 10/2 х 1 1/2 = 10/2 х 3/2 = (10 х 3)/2 = 30/2 = 15.

        Если говорить проще, то в данном случае необходимо привести все к общему знаменателю для того, чтобы выполнить дальнейшие действия умножения дробей. Если возникают какие-либо трудности с самостоятельным решением подобных примеров, тогда лучше воспользоваться онлайн-калькулятором, что позволит существенно упростить операцию, а также уменьшить временные затраты при решении более сложных и трудоемких примеров.

        Как умножать 3 и более дробей?

        Если необходимо умножить три и более дробей, можно воспользоваться соответствующим правилом. Числитель будет равен произведению этих чисел, а знаменатель – произведению соответствующих знаменателей дробей. Умножение можно выполнить по формуле, поэтому достаточно следовать указанным правилам. В качестве примера можно рассмотреть следующее:

        2/3 х 1/5 х 3/7 = (2 х 1 х 3)/(3 х 5 х 7) = 2/35.

        Как видно, умножение такого рода достаточно простое, поэтому не требует никакого труда и усилий. Также можно легко воспользоваться специальным калькулятором для того, чтобы провести расчеты. Это поможет упростить задачу максимально и снизить временные затраты.

        Правила умножения дробей обычно простые, поэтому каждый сможет с ними справиться. Для выполнения операции достаточно понимать базовые принципы выполнения транзакции. Также можно воспользоваться калькулятором для получения результатов и выполнения более простых математических вычислений.

        Онлайн-калькулятор умножения дробей поможет существенно упростить многие задачи для любого студента, ученика или преподавателя, которому нужно быстро проверить тот или иной пример. Теперь больше не нужно задумываться о сложных операциях или самостоятельно придумывать какие-либо правила. Можно быстро воспользоваться всеми преимуществами, которые предлагаются.

        Вычитание дробей с разными знаменателями · Калькулятор Онлайн

        Калькуляторы онлайн/ Решение уравнений/ Упрощение выражений/ Дроби/ Вычитание дробей

        Учитель очень удивится увидев твоё верное решение😉

        Шаг 1.

        2
        Функция — Квадрат x
        ctg(x)
        Функция — Котангенс от x
        arcctg(x)
        Функция — Арккотангенс от x
        arcctgh(x)
        Функция — Гиперболический арккотангенс от x
        tg(x)
        Функция — Тангенс от x
        tgh(x)
        Функция — Тангенс гиперболический от x
        cbrt(x)
        Функция — кубический корень из x
        gamma(x)
        Гамма-функция
        LambertW(x)
        Функция Ламберта
        x! или factorial(x)
        Факториал от x
        DiracDelta(x)
        Дельта-функция Дирака
        Heaviside(x)
        Функция Хевисайда

        Интегральные функции:

        Si(x)
        Интегральный синус от x
        Ci(x)
        Интегральный косинус от x
        Shi(x)
        Интегральный гиперболический синус от x
        Chi(x)
        Интегральный гиперболический косинус от x

        В выражениях можно применять следующие операции:

        Действительные числа
        вводить в виде 7. 3
        — возведение в степень
        x + 7
        — сложение
        x — 6
        — вычитание
        15/7
        — дробь

        Другие функции:

        asec(x)
        Функция — арксеканс от x
        acsc(x)
        Функция — арккосеканс от x
        sec(x)
        Функция — секанс от x
        csc(x)
        Функция — косеканс от x
        floor(x)
        Функция — округление x в меньшую сторону (пример floor(4.5)==4.0)
        ceiling(x)
        Функция — округление x в большую сторону (пример ceiling(4.5)==5.0)
        sign(x)
        Функция — Знак x
        erf(x)
        Функция ошибок (или интеграл вероятности)
        laplace(x)
        Функция Лапласа
        asech(x)
        Функция — гиперболический арксеканс от x
        csch(x)
        Функция — гиперболический косеканс от x
        sech(x)
        Функция — гиперболический секанс от x
        acsch(x)
        Функция — гиперболический арккосеканс от x

        Постоянные:

        pi
        Число «Пи», которое примерно равно ~3. 14159..
        e
        Число e — основание натурального логарифма, примерно равно ~2,7183..
        i
        Комплексная единица
        oo
        Символ бесконечности — знак для бесконечности

        Калькулятор умножения дробей

        Ресурсы Живой конкурс Практика Проблемы Математические задачи Пошаговая работа Мгновенный математический решатель Создатель задач в классе Школы рядом с вами

        Академики КГ 1 класс 2 класс 3 класс 4 класс 5-й класс 6 класс

        • Академики
        • Ресурсы
        • Безопасность
        • Цены
        • регистр
        • Авторизоваться

        1-й КЛАСС2-й КЛАСС3-й КЛАСС4-й КЛАСС5-й КЛАСС6-й КЛАСС

        Сложение ДробейВычитание ДробейУмножение ДробейДробное ДелениеОбратное числоДней между двумя датамиДлительность времениУпростить отношениеЭквивалентные отношенияНайти пропущенные в пропорцииПроверить пропорции отношений ionЗаполните таблицу соотношенийАнализ размеровПлощадь (правильные формы)ПериметрLCMGCFLCDНайти LCM из GCFНайти GCF из LCMДесятичный Преобразование в дробиПреобразование в десятичные числаУмножение целых чиселРазложение десятичных чиселРазложение десятичных чисел в словахВычитание нескольких десятичных знаковПриведение значения к десятичному Десятичное в месте значенийОкругление десятичного числаУпрощение числового выраженияАрифметика дробейАрифметика смешанных чиселПроверка високосного годаПреобразование дробей и смешанных чиселПреобразование обычных единиц измеренияDraw Bar Graph

        Фракция А

        Фракция В

        5/8 х 2/3 = 5/12

        показать тренировку

        Вопрос:
        5/8 х 2/3 = ?
        Ответ:

        5/8×2/3 «=»
        Умножить оба числителя и умножить оба знаменателя
        5/8×2/3 «=» 5 х 28 х 3 =10/24

        Упростите полученную дробь с помощью НОД
        Найдите НОД для числителя и знаменателя
        НОД(10, 24) = 2
        Разделить числитель и знаменатель на НОД
        1024 =10 ÷ 224 ÷ 2 =512
        5/8×2/3 = 5/12

        Калькулятор умножения дробей, показывающий работу по нахождению произведения двух одинаковых или разных дробей. Пошаговое вычисление помогает родителям помочь своим детям, учащимся 4, 5 или 6 класса, проверить работу и ответы домашних заданий на умножение одинаковых или непохожих дробей и задач по предварительной алгебре или по числам и операциям с дробями (NF). единых базовых государственных стандартов (ОКГС) по математике.

        Связанные калькуляторы

        • Калькулятор сравнения дробей
        • Калькулятор вычитания дробей

        Связанные задачи

        • Добавление дробей
        • Умножение дробей с одинаковыми знаменателями
        • Сложение смешанных и целых чисел

        Связанные рабочие листы

        • Сравните и найдите большую дробь
        • Сложение для дробей с одинаковыми знаменателями
        • Вычитание дробей с одинаковыми знаменателями
        • Умножение дробей с одинаковыми знаменателями
        • Упорядочивание дробей от наименьшего к наибольшему
        • Порядок убывания дробей с одинаковыми знаменателями
        • Сравните и найдите большую дробь
        • Сложение дробей с одинаковыми знаменателями
        • Вычитание дробей с одинаковыми знаменателями
        • Умножение дробей с одинаковыми знаменателями
        • Упорядочивание дробей от наименьшего к наибольшему
        • Порядок убывания дробей с одинаковыми знаменателями
        • Добавление дробей
        • Умножение дробей с одинаковыми знаменателями
        • Сложение смешанных и целых чисел

        Калькулятор умножения дробей и решатель

        Получите подробные решения ваших математических задач с помощью нашего

        Пошаговый калькулятор умножения дробей . Практикуйте свои математические навыки и учитесь шаг за шагом с помощью нашего математического решателя. Проверьте все наши онлайн-калькуляторы здесь!

        1

        2

        3

        4

        5

        6

        7

        8

        900 02 9

        a

        b

        c

        d

        f

        g

        m

        n

        u

        v

        w

        x

        г

        г

        .

        (◻)

        +

        ×

        ◻/◻

        /

        ÷

        ◻ 90 172 2

        √◻

        √ ◻

        e

        π

        ln

        лог

        лог 90 202 ◻

        lim

        d/dx

        D x

        |◻|

        θ

        =

        >

        <

        >=

        <=

        sin

        cos

        tan

        кроватка

        sec

        csc

        asin

        acos

        atan

        acot

        asec

        acsc

        sinh

        cosh

        tanh

        coth

        sech

        csch

        асинх

        акош

        атан

        акот

        асеч

        аш

        Пример

        Решенные проблемы

        Сложные задачи

        1

        Решенный пример перекрестного умножения дробей

        $\frac{2x-9}{10}+\frac{2x-3}{2x-1}=\frac{x}{5}$

        2

        Наименьшее общее кратное (НОК) суммы алгебраических дробей состоит из произведения общих множителей с наибольшим показателем степени и необычных множителей

        $L.

        Бесплатные игры в шахматы онлайн: Игры шахматы — играть онлайн бесплатно на сервисе Яндекс Игры

        Игра Шахматы: онлайн 2 игрока (Chess: Online 2 Player)

        На Двоих Шахматы Семейные Логические

        106

        13

        HOT

        HOT

        • Hot Chess Master King

          4,4 1036

        • Hot Two Player Chess

          4,2 491

        Похожие Шахматы игры

        • New Chess Mr

          5,0 63

        • New Chess — Clash of Kings

          4,8 215

        • New Chess Game Jigsaw

          4,8 21

        • New Chess Master King

          4,4 1036

        • New Chess Move 2

          4,4 16

        • New Chess Classic

          4,4 94

        • Hot Master Chess

          4,9 3359

        • Hot Checkers Dama chess board

          4,5 1757

        • Hot Chess Master King

          4,4 1036

        • Hot Two Player Chess

          4,2 491

        • Hot Master Chess Multiplayer

          4,5 349

        • Hot Chess Classic: 2 Player Game

          4,7 246

        • Hot Chess — Clash of Kings

          4,8 215

        • Hot Master Chess: 2 Player

          4,4 196

        • Hot Ultimate Chess

          4,8 193

        • Hot Real Chess Online

          4,6 158

        • Hot 3D Chess

          4,5 145

        • Hot Chess: Online 2 Player

          4,5 145

        Другие Спортивные игры

        • New Football Head Sports — Multiplayer Soccer Game

          4,9 20

        • New Become a referee

          4,3 82

        • New Football 3D

          5,0 269

        • New Goalkeeper Wiz

          4,3 24

        • New Minitoss

          4,3 24

        • New Soccer Free Kick

          4,5 123

        • Hot Head Ball — Online Soccer Game

          4,1 28014

        • Hot Head Ball 2 — Online Soccer Game

          4,3 9392

        • Hot Soccer Stars Classic

          4,3 5847

        • Hot Soccer Heads 2

          4,2 3747

        • Hot Rocket Soccer Derby

          4,1 2500

        • Hot Football Strike — Multiplayer Soccer

          4,2 1739

        • Hot Toon Cup 2022

          4,2 1681

        • Hot Dream Head Soccer

          4,5 1574

        • Hot Champion’s Field: Football

          4,3 1228

        • Hot Head Ball Soccer

          4,7 1097

        • Hot KiX Dream Soccer

          4,4 1041

        • Hot Car Eats Car 4

          4,4 1030

        Game information

        Game name

        Шахматы: онлайн 2 игрока

        Оригинальное имя

        Chess: Online 2 Player

        Played

        145

        Лайков

        106

        Доступно

        PC, Mobile

        Добавлен

        07. 12.2019

        Языки

        Русский

        Мерсибо. Развивающие игры для детей, логопедов, педагогов, психологов

        Портал для детских специалистов

        Материалы для развивающих и коррекционных занятий, обследования детей, создания пособий и повышения квалификации

        Для логопедов, воспитателей, дефектологов и других детских специалистов

        Для занятий с детьми от 2 до 9 лет

        Для проведения обследований и занятий с детьми и повышения квалификации специалистов

        Зарегистрируйтесь и получите доступ к 10 онлайн-играм

        Для логопедов, дефектологов,
        психологов и педагогов

        Сложно заинтересовать детей
        традиционными методиками?
        Ищете новые подходы для
        работы с современными детьми?

        Хотите, чтобы дети с удовольствием
        посещали ваши занятия и быстрее
        добивались результата?

        Тратите много времени на подготовку
        к занятиям и домашние задания? Нет
        времени на свою семью и отдых?

        Хотите моментально создавать
        речевые карты, рабочие
        программы и индивидуальные
        маршруты для отчетности?

        Хотите первыми узнавать
        о передовом опыте и лучше
        разбираться в уже
        существующих методиках?

        Не знаете, как снизить эмоциональную
        нагрузку на работе и избежать
        профессионального выгорания?

        МЕРСИБО — уникальный портал, созданный специалистами для специалистов

        Начинающие специалисты

        Получите надежный старт в карьере, сделайте
        первые шаги вместе с Мерсибо

        1. Освойте инструменты экспресс-обследования речевого развития ребенка
        2. Приобретите качественные настольные пособия, уникальные игры на флешках и программы для повседневной работы
        3. Смотрите вебинары по организации работы: как подобрать методику, провести обследование, организовать коррекционные и развивающие занятия, как подготовить отчетность и привлечь родителей
        4. Получите подборку из золотого фонда литературы и проверенных методик от наших спикеров

        Профессионалы

        Откройте новые грани профессии благодаря
        интерактивным методикам и обучению от Мерсибо

        1. Освойте современные методики и новые подходы от ведущих специалистов для более продуктивных занятий
        2. Повысьте свою квалификацию и заработок после прохождения наших вебинаров и курсов
        3. Используйте практический материал для ежедневной работы
        4. Экономьте на дидактических материалах и обучении с нашим порталом

        Интерактивные игры для развивающих и коррекционных занятий для детей 2-10 лет

        Бесплатная библиотека пособий

        Пособие для развития логики и мышления
        «Бабушка на кухне»

        Пособие для знакомства с окружающим миром
        «Тропический лес»

        Пособие для работы над звуко-буквенным анализом
        «Что в начале слова: Б или П?

        Пособие для автоматизации звуков
        «Убери из гаража лишние предметы»

        Интерактивный Конструктор картинок

        Генератор корректурных таблиц

        • Формируйте корректурные таблицы любой сложности за пару минут.
        • Больше не надо искать подходящие задания в магазинах — готовьте их сами за 5 минут.
        • Создание таблиц интуитивно понятно, вам не потребуется использовать офисные программы.
        • 150 готовых пособий от логопеда и нейропсихолога, специализирующегося на вопросах дисграфии и дислалии.
        • Можно заниматься в интерактивном формате, без распечатывания заданий. Экономьте бумагу и чернила.

        Подробнее о Генераторе

        Редактор — Наталья Викторовна Микляева

        Программу подготовил коллектив педагогов под редакцией Натальи Викторовны Микляевой.

        Наталья Микляева — профессор, член Международной аттестационной палаты. Регулярно участвует в форумах по управлению качеством коррекционно-развивающего процесса и проводит мастер-классы по ведению документации.

        О Конструкторе

        Экспресс-обследование речевого развития ребенка

        Бесплатные вебинары для специалистов

        Узнавайте о современных
        методиках и подходах в своей
        профессии, не выходя из дома

        Эффективное развитие лексической базы у детей с ЗПР и ОНР с помощью компьютерных игровых технологий

        Суслова Екатерина Александровна

        ПодробнееЗарегистрироваться

        #логопед  #дефектолог 

        Узнавайте о современных
        методиках и подходах в своей
        профессии, не выходя из дома

        1. Учитесь у лучших специалистов страны
        2. Посещайте живые вебинары и задавайте вопросы лекторам
        3. Смотрите записи вебинаров в удобное вам время
        4. Получайте бесплатные сертификаты после вебинаров

        Перейти к вебинарам

        Курсы повышения квалификации
        и мастер-классы

        Нейропсихологические и логопедические технологии развития речи у детей

        Ланина Татьяна Николаевна

        ПодробнееЗарегистрироваться

        #воспитатель  #дефектолог  #учитель начальных классов  #методист 

        Программа работы по выходу неговорящего ребенка из безречья к развернутой фразовой речи

        Бунина Виктория Станиславовна

        ПодробнееЗарегистрироваться

        #логопед  #дефектолог 

        Воспитание и обучение детей с ЗПР: современные технологии работы

        Микляева Наталья Викторовна

        ПодробнееЗарегистрироваться

        #логопед  #воспитатель  #дефектолог  #психолог  #учитель начальных классов  #методист 

        Выбрать интересные

        Магазин для коррекционных педагогов

        Хотите пользоваться продуктами
        Мерсибо на выгодных условиях?
        Оформите подписку специалиста

        Доступ к 337+ играм для коррекционных
        и развивающих занятий

        «Конструктор картинок» для создания
        собственных игр и пособий

        Личный кабинет для ведения
        документации и дневника занятий с детьми

        Библиотека пособий
        с возможностью изменять пособия

        Программа экспресс-обследования
        речевого развития ребенка

        Уведомления о предстоящих вебинарах,
        курсах, новых продуктах, скидках и акциях

        При покупке годовой подписки вы получаете
        сертификат активного пользователя ИКТ

        Подробнее о подписке

        Зарегистрируйтесь на портале

        и оцените качество и методическую проработку наших игр

        1. Игры подходят для коррекционных и развивающих занятий
        2. Индивидуальных и групповых
        3. С детьми разного возраста и разными нарушениями
        4. В ДОУ, развивающих и коррекционных центрах, начальных классах школ, домах ребенка, центрах реабилитации и для частной практики
        5. Дома с родителями или самостоятельно
        6. После регистрации вы получите доступ к 10-ти играм портала

        Бесплатные онлайн-игры в шахматы

        Один из лучших способов узнать о шахматах и ​​улучшить свою игру в шахматы — это смотреть или участвовать в настоящих шахматных партиях, особенно в партиях лучших шахматистов. Этот раздел базы знаний ChessCentral предлагает множество бесплатных онлайн-шахматных игр с поучительными комментариями мастеров шахмат или самих игроков. Эти шахматные партии относятся к самым ранним шахматным временам вплоть до наших дней, и каждая онлайн-игра в шахматы предлагает элементы шахматной стратегии или шахматной дебютной игры, которые принесут пользу каждому прогрессирующему шахматисту. Что еще более важно, эти бесплатные шахматы являются прекрасными примерами шахматного искусства в действии!

         

        Чудак
        Если белые намерены сыграть 1.f4, дебют Берда, они могут довольно часто встречать гамбит Фрома ходом 1…e5. Эта популярная атака гарантирует захватывающую игру со множеством сложных обходных путей, и оба игрока должны знать немного теории, чтобы сражаться на равных. Но если белые могут с самого начала направить игру на неизведанную территорию — неизведанную для их противника! — тогда его шансы на успех намного повышаются.

        Подробнее

        Необычная концовка
        Одна из первых вещей, которую изучает каждый шахматист, — это основные маты. Король и ферзь против короля, ладья и король против короля — и так далее до пары король плюс слон против короля, и сложный король со слоном и конем, матовым королю. Однако многие хорошие игроки совершенно не знакомы с последним «базовым» матом, когда король плюс два коня против короля и пешки. Даже специалисты по шахматам, знающие об этом сложном мате, часто не умеют его исполнять. Подробнее


        Случайное число Фишера в 1875 году
        Даже «Случайное число Фишера» Бобби, также известное как «Шахматы960», не лишено прецедентов в наших стандартных шахматах. Мы знаем, что Фишер восхищался Стейницем и был хорошо знаком с партиями и сочинениями первого чемпиона мира. Подростком Фишер был замечен за чтением журнала International Chess Magazine, а совсем недавно, в 1996 году, он был замечен за покупкой коллекции партий Стейница в шахматном магазине в Аргентине. Можно догадаться, знал ли Фишер о следующей игре, сыгранной зимой 1875 года между Блэкберном и Поттером, или о других, подобных этой. Было бы интересно собрать примеры случайных чисел Фишера до Фишера, чтобы посмотреть, практиковались ли другие аранжировки частей. Стейниц, из «Поля» (октябрь 1875 г.): Подробнее

         

        Вена-Париж Почтовые шахматы
        Следующая партия является одной из двух, аннотированных Вильгельмом Стейницем в январском номере журнала International Chess Magazine за 1886 год. Здесь мы видим Стейница за работой шахматного журналиста, сталкивающегося с дедлайнами и другими трудностями, но при этом обращаем внимание на тщательность и тщательность его анализа. Обе игры представлены в формате ChessBase для загрузки на веб-странице, указанной на компакт-диске «Собрание сочинений Вильгельма Стейница», полную информацию о котором можно найти там. Таким образом Стейниц аннотировал сотни партий, которые вместе составляют основу современных шахмат. Читать далее.

         

        Steinitz-Zukertort, 1886
        Эта статья представляет собой образец того, как Вильгельм Стейниц освещал Первый чемпионат мира, и представлена ​​для вашего удовольствия из Собрания сочинений Вильгельма Стейница. Знакомство с матчем Стейниц-Цукерторт позволит вам ощутить волнение, царившее в начале первого матча на первенство мира. Сама игра богато аннотирована Стейницем и показывает, почему он считается величайшим шахматным инструктором в мире. В этом отрывке представлена ​​лишь одна из сотен партий (более 850), аннотированных человеком, который был кумиром Бобби Фишера и сделал шахматы такими, какие они есть сегодня! А теперь Вильгельм Стейниц Подробнее

         

        Турнир с гандикапом
        Активная шахматная жизнь Лондона в 1870-х годах давала множество материалов для шахматной колонки The Field, журнала, называвшего себя «Газетой деревенского джентльмена». Стейниц редактировал шахматную колонку, представляя постоянный поток шахматных задач, новостей и партий с первоклассными аннотациями. Подробнее

         

        Дебют Сокольского

        Ранее мы узнали, что А. Алехин назвал 1.b4 «старым ходом» в своем комментарии 1924 года к партии Тартаковера-Мароци. Может, и старый, но мало играл в топ-турнирах. Есть игра Хантингтон-Кемени, Нью-Йорк 1891, а затем упомянутая выше игра Тартаковера; с тех пор 1.b4 уделяется очень мало внимания, пока сам Сокольский не победил этим ходом Сало Флор (Москва, 1953). Читать далее.

         

        Бегемот, кто-нибудь?
        Многие шахматисты хотели бы научиться универсальному шахматному дебюту, особенно дебюту, который может быть развернут любым цветом, белым или черным, почти независимо от хода противника. Только несколько шахматных дебютов подходят под это описание, возможно, староиндийская формация и несколько других. Одним из кандидатов на роль такой «универсальной» системы является дебют «Гиппопотам», пожалуй, самый неправильный из нестандартных шахматных дебютов. Подробнее

         

        Необычный шахматный гамбит
        Если белые хотят сыграть дебют «Берд-Ларсен», они чаще всего избегают очень прямого порядка ходов 1.f4 d5 2.b3. Потому что у черных есть шанс пересечь эту схему ранним ходом …d5-d4, как в партии ниже. Если первый игрок откладывает b2-b3 в пользу развития королевского фланга, то черные могут сами фианкетто ходом . ..g7-g6, чтобы помешать плану белых. В любом случае, те, кто играет в Дебют Берда, нашли способы маневрировать вокруг этого рывка …d5-d4, но и агрессивный гамбитный вариант тоже приветствуется! Подробнее

         

        Игра в шахматы, в которую вы не поверите
        Один из аспектов шахматных исследований, который мне очень нравится, заключается в том, что вы никогда не знаете, куда это вас приведет. На днях, занимаясь исследованиями для книги, которую я пишу, я наткнулся на действительно крутую маленькую странность: партию из сеанса Хосе Капабланки 1924 года, в которой великий человек на самом деле проиграл любителю менее чем за двадцать ходов. Подробнее

         

        Обезьянье дело в шахматах
        Если первый игрок хочет пошалить в шахматном дебюте, он может сыграть 1.b4, дебют орангутана. Мы знаем, что Савелли Тартаковер назвал этот дебют после «консультации» с орангутангом в зоопарке Бронкса и, соответственно, отважился на 1.b4 на следующий день (21 марта 19 марта).24) против Гезы Мароци. Тартаковер получил прекрасную позицию, но, не по вине мудреца-обезьяны, одержал только упорную ничью. Читать далее

         

        КГД, полугрюнфельдский дебют
        Мы знаем из шахматных дебютов, как защита Грюнфельда (1.d4 Кf6 2.c4 g6 3.Кc3 d5), что черные могут контролировать центр фигурами вместо пешек. Но как насчет бедного кузена Грюнфельда, 1.d4 d5 2.c4 g6, явно члена этой сверхсовременной семьи? В первом выпуске Kamikaze Times (ноябрь 2002 г.) этот вариант был назван «защитой Алехина» от ферзевого гамбита. Алехин действительно сыграл этот дебют, но редактор правильно отмечает, что Блэкберн имеет преимущество. Необычный и редко встречающийся, здесь не так много теории для изучения и не так много игр для консультаций; те, кому нравятся необычные шахматные дебюты, могут продолжить расследование. Сначала у нас работает Блэкберн: Подробнее

         

        Доктор Рон играет с птицей!
        Играть 1.f4 белыми? Но я всю жизнь играю 1.е4, с тех пор как в три года научился шахматам у своего отца. Единственными партиями, в которые мы играли, были дебюты с двойным королем и ходом 1.e4 e5. В 70-х годах я шесть лет учился у старого мастера Дона Брукса (многократного чемпиона штата Индиана). Его правилом для меня было: «1.e4 — если это достаточно хорошо для Бобби Фишера, это достаточно хорошо и для вас». Ruy Lopez был его выбором для меня. После первого года у него я изучил партии Фишера, Бронштейна, Таля, Кереса и Алехина. Подробнее

        Стейниц в Новом Орлеане
        Местные газеты жадно писали о путешествиях Вильгельма Стейница, и его поездка в Новый Орлеан в 1883 году не стала исключением. Там городской газетой была Daily Picayune, которая рассказывала о поездке Стейница в Новый Орлеан на новогодние праздники 1883 года. Мы знаем, что Стейниц дал сеанс одновременной игры 2 января, из которого 526-я партия в «Собрании сочинений» единственный пример. Подробнее

        Матч Стейниц-Мартинес, 1883
        Местные и региональные газеты все чаще публикуют записи о шахматных партиях Вильгельма Стейница. Шахматные колонки в любой городской газете записывали визит чемпиона, часто приводя результаты игр с любых выступлений, проводимых Стейницем. Эти бумаги нужно только прочитать. Подробнее

        Знаменитый неудачник
        С удовольствием представляем вам «Знаменитый неудачник» историка шахмат Томаша Лиссовского и гроссмейстера Бартломея Мачеи. Написанная специально для ChessCentral, эта важная статья дает многим из нас первое представление о жизни и творчестве Лайонела Кизерицкого — возможно, самого известного «неудачника» в анналах шахмат. Однако, как мы увидим, Кизерицкий был гораздо большим; он действительно был первоклассным шахматистом, уступавшим в свое время только Андерссену и (вероятно) Стонтону. Подробнее

        Горстка Бессмертия
        Почти у всех шахматистов в послужном списке есть личные «Бессмертные» партии. Известны и национальные «Бессмертные» игры — например, «Русская бессмертная игра» (Шишкин-Гриксберг, СПб. 1889 г.) и «Польская бессмертная игра» (Гриксберг-Найдорф, Польша, 1935 г. ) и многие другие. Чарушин собрал особенные Immortal Games! Подробнее

        A Morphy Curiosity
        Ниже приводится «возможная реставрация» неизвестной игры Morphy, предоставленной г-ном А. Г. Селлманом и опубликованной в выпуске № 1 журнала The International Chess Magazine (январь 1885 г.) под редакцией Стейница. Что-то вроде ретроградного анализа, вроде реверс-инжиниринга шахматной позиции! Подробнее

         

        Шахматная машина

        Следующая партия была сыграна в конце 1918 года Капабланкой белыми против Яновски на черной стороне. Кубинец уже был широко признан лучшим шахматистом мира, а Капабланка официально получил титул чемпиона мира по шахматам в 1921 году после победы над Ласкером. На самом деле Капабланка в этот период играл настолько убедительно, что в качестве предварительного условия матча Ласкер настоял на отказе от титула, заявив в своем соглашении (27, 19 июня20): «Вы заслужили этот титул не формальностью вызова, а своим блестящим мастерством». Подробнее

         

        Познакомьтесь с герром Лёвенталем

        Одним из «старых мастеров» шахмат досовременной эпохи является Иоганн Якоб Лёвенталь, имя которого сегодня в значительной степени забыто или помнят, если вообще помнят, из-за поражения в шахматном матче против Пола Морфи. . Напомним, что после того, как молодой Морфи представился победой на Первом американском шахматном конгрессе (ноябрь 1857 г.), он отправился в европейское турне, прибыв в июне 1858 г. в Лондон. Там Морфи не смог договориться о матче с Ховардом Стонтоном, и после нескольких выездных и показательных игр с местными экспертами в июле был начат матч с Левенталем. Конечным результатом конкурса стала убедительная победа Морфи, набравшего 9 баллов.выигрывает при 3 поражениях и 2 ничьих. Подробнее

         

        «Шаблон» в шахматах

        Каждому шахматисту, который хочет улучшить свою игру, со временем понадобится специальная группа шахматных дебютных позиций, определенное количество ключевых позиций, которые он будет знать лучше, чем кто-либо другой. Не просто дебютный вариант, а таблица или шаблон — расстановка фигур и пешек, в которой игрок чувствует себя абсолютно комфортно и понимает различные возможности этой позиции. Предприимчивый шахматист попытается раскрыть каждый нюанс принятой им картины, осваивая сначала одну такую ​​ключевую позицию, а затем добавляя другие. Некоторые игроки будут экспертами в дюжине шаблонов; другие будут использовать только четыре или пять; некоторые будут специализироваться на гамбитах, другие — на стратегических паттернах. Подробнее

         

         

        Скачать онлайн-шахматы — лучшее программное обеспечение и приложения

        Реклама

        1. 3D Chess Unlimited — это видеоигра для операционной системы Microsoft Windows, которая позволяет игрокам бросить вызов компьютеру или сетевому противнику. ..

        2. Настоящие шахматы онлайн — это шахматная игра, в которую можно играть на ПК и устройствах Android. Эта игра предлагает 2400 уровней искусственного интеллекта и множество игровых режимов. Есть…

        3. 5D Chess — настольная игра, созданная независимыми разработчиками Конором Петерсеном и Thunkspace LLC. Правила игры такие же, как и в обычных шахматах. За исключением…

        4. Grand Master Chess Online — потрясающая пробная версия игры, доступная только для Windows, которая относится к категории игр для ПК с подкатегорией Шашки и…

        5. Один из способов убить скуку — учиться и играть в шахматы с семьей или друзьями. Загрузите это приложение бесплатно, и вы обязательно получите удовольствие. Это настольная игра…

        6. шахматы Challenger — отличная альтернатива, если у вас нет физической доски. Два игрока могут соревноваться в виртуальной среде и как…

        7. Это приложение является лучшим местом, чтобы играть в бесплатные онлайн шахматы. Вы можете играть против других игроков по всему миру или локально со своими друзьями. Есть несколько игр…

        8. lichess — это бесплатная онлайн-игра в шахматы с различными вариантами, включая различные стили доски, темы, режимы игры, игроков и многое другое. Вы можете играть в шахматы онлайн…

        9. Шахматы всегда были любимой игрой, требующей логической стратегии, чтобы победить противника. Если вы любитель шахмат и хотите развиваться дальше…

        10. Играйте в живые шахматы бесплатно! Это приложение позволяет вам играть против людей со всего мира. Вы можете выбирать из разных шахматных фигур и играть в местные игры…

        11. С LiKeyChess вы можете играть в шахматы на Lichess.org с помощью клавиатуры! Это очень полезное приложение для тех, кто хотел бы играть в шахматы на своем рабочем столе. …

        12. Бесплатная и простая игра в шахматы теперь доступна онлайн. С помощью этого приложения вы можете поставить мат своему противнику всего за несколько ходов, даже если вы…

        13. Шахматы (онлайн и 960), разработанные CP Abdelhady, переносят на ваше мобильное устройство традиционную игру в шахматы — так что вы можете наслаждаться игрой в виртуальной.

        Log4(x+5)=Log4(4x-10) — Знания.site

        Последние вопросы

        • Математика

          35 секунд назад

          Запиши десятковий дріб, який має: 15 цілих 12 тисячних;​
        • Математика

          36 секунд назад

          Допоможіть будь ласка 1) 62,12-61,44:1,22) 35:0,5-(3-0,594):0,153) 10,8:(7,67-1,4•2,05)4) 0,0432:0,12+0,2:0,255) (1:0,08-9,75):0,0256) 10,79:8,3-(5-0,56):3,7
        • Математика

          36 секунд назад

          У якій точці пряма 3х + 2у = 6 перетинає вісь Ох?​
        • Математика

          36 секунд назад

          Відстань між двома містами 105 км. Одночасно з цих міст назустріч один одному виїхали два велосипедисти і зустрілись через 3,5 год. Знайди швидкість кожного ве- лосипедиста, якщо швидкість першого була на 2 км/год більша, ніж швидкість другого.Напишіть, умову, що писати у дужках та розв’язання ​. дякую
        • Математика

          5 минут назад

          У п’ятницю Тетянка прочитала 5 сторінок книги. Кожного наступного дня вона читала вдвічі більше, ніж попереднього дня. У який день тижня Тетянка дочитає книгу, якщо в ній 35 сторінок?методом схематичного малюнку​
        • Математика

          5 минут назад

          Помогите решить задачу Швидкість автомобіля дорівнює 70км/год.Швидкість мотоцикліста становить 0,6 від швидкості автомобіля, а швидкість велосепедиста-0,1 від швидкості мотоцикліста. Скільки кілометрів проїде велосепидіст за 2 години?
        • Математика

          5 минут назад

          в киви курс валют 100 тенге — 16,20₽, а 1₽ сколько будет в тенге? Распишите как рассчитать​
        • Математика

          10 минут назад

          Будь ласка допоможіть
        • Математика

          10 минут назад

          Округлення до десятків 7894,25486​
        • Математика

          10 минут назад

          -12, 5+8 1/5 срочно пжпжпж
        • Математика

          15 минут назад

          за 4 морозива заплатили 32 грн. скільки потрібно заплатити за 7 штук такого морозива?​
        • Математика

          15 минут назад

          4. Побудуйте графік функції y = |logg(x — 1)|​
        • Математика

          20 минут назад

          познач рисунок на якому, зафарбовано 1/2 променів сонечка. ​
        • Математика

          20 минут назад

          Скільки коронів має рівняння 2х :0=0
        • Математика

          20 минут назад

          Помогите срочно а) -9 • 4 • (-4) • (-25) Б) -12,5 • 4 • (-8) • (-5)

        Все предметы

        Выберите язык и регион

        English

        United States

        Polski

        Polska

        Português

        Brasil

        English

        India

        Türkçe

        Türkiye

        English

        Philippines

        Español

        España

        Bahasa Indonesia

        Indonesia

        Русский

        Россия

        How much to ban the user?

        1 hour 1 day 100 years

        3-8 9 Оценить квадратный корень из 12 10 Оценить квадратный корень из 20 11 Оценить квадратный корень из 50 94 18 Оценить квадратный корень из 45 19 Оценить квадратный корень из 32 20 Оценить квадратный корень из 18 92

        Решите log4(x+4) – 2 log4(x+1) =1/2 для x.

        Цитата страницы Начать эссе значок-вопрос Спросите репетитора

        Начать бесплатную пробную версию

        Скачать PDF PDF Цитата страницы Цитировать Поделиться ссылкой Делиться

        Ссылайтесь на эту страницу следующим образом: 92+3x-2=0`


        `(2x-1)(x+2)=0`

        Итак, `x=1/2` или `x=-2` . Но -2 не находится в домене исходной функции.

        Таким образом, решение `x=1/2` .

        См. eNotes без рекламы

        Запустите 48-часовую бесплатную пробную версию , чтобы получить доступ к более чем 30 000 дополнительных руководств и более чем 350 000 вопросов помощи при выполнении домашних заданий, на которые наши эксперты ответили.

        Получите 48 часов бесплатного доступа

        Уже зарегистрированы? Войдите здесь.

        Утверждено редакцией eNotes

        Математика

        Последний ответ опубликован 07 сентября 2010 г. в 12:47:25.

        Что означают буквы R, Q, N и Z в математике?

        14 Ответы педагога

        Математика

        Последний ответ опубликован 09 октября 2017 г. в 12:54:39ЯВЛЯЮСЬ

        Добавьте 1 плюс 2 плюс 3 плюс 4. . . вплоть до 100.

        3 Ответы воспитателя

        Математика

        Последний ответ опубликован 25 февраля 2016 г. в 18:48:45.

        Сколько времени (в часах) займет ваше путешествие, если вы проедете 350 км со средней скоростью 80 км/ч? Какова формула с данными: время, расстояние, скорость или скорость?

        1 Ответ воспитателя

        Математика

        Последний ответ опубликован 3 октября 2011 г.

      Найти периметр прямоугольника abcd 3 класс: 3 класс. Рабочая тетрадь №1. Ответы к стр. 18

      ГДЗ по геометрии 8 класс Атанасян. Гл.V №401. Найдите периметр прямоугольника ABCD… – Рамблер/класс

      ГДЗ по геометрии 8 класс Атанасян. Гл.V №401. Найдите периметр прямоугольника ABCD… – Рамблер/класс

      Интересные вопросы

      Школа

      Подскажите, как бороться с грубым отношением одноклассников к моему ребенку?

      Новости

      Поделитесь, сколько вы потратили на подготовку ребенка к учебному году?

      Школа

      Объясните, это правда, что родители теперь будут информироваться о снижении успеваемости в школе?

      Школа

      Когда в 2018 году намечено проведение основного периода ЕГЭ?

      Новости

      Будет ли как-то улучшаться система проверки и организации итоговых сочинений?

      Вузы

      Подскажите, почему закрыли прием в Московский институт телевидения и радиовещания «Останкино»?

      Нужна услуга, не могу разобраться, задача Гл. V №401. 
       Найдите периметр прямоугольника ABCD, если биссектриса угла А делит сторону: а) ВС на отрезки 45,6 см и 7,85 см;
      б)  DC на отрезки 2,7 дм и 4,5 дм.
       
       

      ответы

      Я разобралась. Вот ответ на задачу Гл.V №401


      1. BE = 45,6 см, BС = 7,85 см => PABCD =
      = 2(АВ + ВС) = 2(АВ + BE + EС) = 2(45,6 + 45,6 +
      + 7,85) = 198,1 см.
      2. BE = 7,85 см, EС = 45,6 => PABCD =
      = 2(AВ + ВС) = 2(AB + BE + ВС) = 2(7,85 + 7,85 +
      + 45,6) = 122,6 см.
       
      б) 

      1 . DЕ = 2,7 дм, ЕС = 4,5 => РABCD = 2(АD+
      + DС) = 2(АD + DЕ + ЕС) = 2(2,7 + 2,7 + 4,5) = 19,8 дм.
       2 . DЕ = 4,5 дм, ЕС = 2,7 => РАВСD = 2(АD+
      + DС) = 2(АD + DЕ + ЕС) = 2(4,5 + 4,5 + 2,7) = 23,4 дм.

      Ответ: а) 198,1 см или 122,6 см; б) 19,8 дм или 23,4 дм.
       

      ваш ответ

      Можно ввести 4000 cимволов

      отправить

      дежурный

      Нажимая кнопку «отправить», вы принимаете условия  пользовательского соглашения

      похожие темы

      Экскурсии

      Мякишев Г. Я.

      Психология

      Химия

      похожие вопросы 5

      Привет! Равносильны ли уравнения? № 55.4 ГДЗ Алгебра 10-11 класс Мордкович.

      Равносильны ли уравнения: (Подробнее…)

      ГДЗАлгебра10 класс11 классМордкович А.Г.

      Найдите координаты № 922 ГДЗ Геометрия 9 класс Атанасян Л.С.

      Найдите координаты вектора ͞a + ͞b, если: а)
        (Подробнее…)

      ГДЗГеометрия9 классАтанасян Л.С.

      ГДЗ по геометрии 8 класс Атанасян. Гл.V №365. Найдите углы А, В и С…

      Нужна помощь. Гл.V №365.
      Найдите углы А, В и С выпуклого четырёхугольника ABCD, если
      (Подробнее…)

      ГДЗАтанасян Л.С.8 классГеометрия

      Найдите периметр № 941 ГДЗ Геометрия 9 класс Атанасян Л.С.

      Найдите периметр треугольника MNP, если М (4; 0), N( 12;-2), P (5;-9).

      ГДЗГеометрия9 классАтанасян Л. С.

      Найдите № 945 ГДЗ Геометрия 9 класс Атанасян Л.С.

      Вершина А параллелограмма ОАСВ лежит на положительной полуоси Ох, вершина В имеет координаты (b; с), а ОА = а. Найдите: а) координаты (Подробнее…)

      ГДЗГеометрия9 классАтанасян Л.С.

      ГДЗ По Математике 5 Класс. Прямоугольник

      Перейти к контенту

      ГДЗ по Математике 5 класс Мерзляк А.Г. § 15. Прямоугольник


      Вопросы к параграфу

      1. Какой четырёхугольник называют прямоугольником?

      2. Какие стороны прямоугольника называют соседними? Противолежащими?

      3. Что называют длиной и шириной прямоугольника?

      4. Каким свойством обладают противолежащие стороны прямоугольника?

      5. Какую фигуру называют квадратом?

      6. Объясните, какие фигуры называют симметричными относительно прямой.

      7. Как называют прямую, относительно которой симметрична фигура?

      8. Какие вы знаете фигуры, имеющие ось симметрии?

      9. Сколько осей симметрии имеет прямоугольник, отличный от квадрата? Квадрат? Равносторонний треугольник


      Решаем устно

      1. Каждая сторона треугольника равна 12 см. Как называют такой треугольник? Чему равен его периметр?

      2. Периметр равнобедренного треугольника равен 32 см, а одна из его сторон — 12 см. Найдите длины двух других сторон треугольника. Сколько решений имеет задача?

      3. Найдите сторону равностороннего треугольника, если она меньше его периметра на 10 см.

      4. Вычислите значение у по формуле у = х • х + 12, если:


      Упражнения

      359. Постройте: 1) прямоугольник, соседние стороны которого равны 4 см и 2 см 2) квадрат со стороной 3 см

      360. Постройте прямоугольник, соседние стороны которого равны 25 мм и 35 мм.

      361. Вычислите периметр: 1) прямоугольника, соседние стороны которого равны 42 см и 23 см 2) квадрата со стороной 8 дм

      362. Найдите периметр прямоугольника, соседние стороны которого равны 13 мм и 17 мм.

      363. Какие из букв, изображённых на рисунке 135, имеют ось симметрии?

      364. Сколько осей симметрии имеет многоугольник, изображённый на рисунке 136?

      365. 1) Длина одной из сторон прямоугольника равна 14 см, что на 5 см больше длины соседней стороны. Найдите периметр прямоугольника. 2) Периметр прямоугольника равен 34 см, а одна из его сторон — 12 см. Найдите длину соседней стороны прямоугольника.

      366. Одна сторона прямоугольника равна 8 см, а соседняя — в 4 раза больше. Найдите периметр прямоугольника.

      367. Квадрат со стороной 12 см и прямоугольник, одна из сторон которого равна 8 см, имеют равные периметры. Найдите неизвестную сторону прямоугольника.

      368. Прямоугольник, соседние стороны которого равны 42 см и 14 см, и квадрат имеют равные периметры. Найдите длину стороны квадрата.

      369. Сколько квадратов изображено на рисунке 137?

      370. Из куска проволоки сделали модель пятиугольника (рис. 138). Какие из моделей перечисленных фигур, длины сторон которых выражаются натуральным числом сантиметров, можно сделать из этого куска проволоки: 1) квадрат; 2) пятиугольник, все стороны которого равны; 3) равносторонний треугольник?

      371. Прямоугольник ABCD разрезали на квадраты так, как показано на рисунке 139. Сторона наименьшего из квадратов равна 4 см. Найдите длины сторон прямоугольника ABCD.

      372. Начертите прямоугольник, соседние стороны которого равны 3 см и 6 см. Разделите его на три равных прямоугольника. Вычислите периметр каждого из полученных прямоугольников. Сколько решений имеет задача?

      373. Существует ли среди прямоугольников с периметром 12 см такой, который можно разделить на два равных квадрата? В случае положительного ответа выполните рисунок и вычислите периметр каждого из полученных квадратов.

      374. Как надо разрезать квадрат на четыре равные части, чтобы из них можно было сложить два квадрата?

      375. Как надо разрезать равнобедренный прямоугольный треугольник на четыре равные части, чтобы из них можно было сложить квадрат?

      376. Как надо разрезать прямоугольник со сторонами 8 см и 4 см на четыре части, чтобы из них можно было сложить квадрат?

      377. Как надо разрезать квадрат на треугольник и четырёхугольник, чтобы из них можно было сложить треугольник?

      378. Как надо разрезать квадрат со стороной 6 см на две части по ломаной, состоящей из трёх звеньев, чтобы из полученных частей можно было сложить прямоугольник?


      Упражнения для повторения

      379. Проведите прямую МК, луч PS и отрезок АВ так, чтобы луч PS пересекал отрезок АВ и прямую МК, а прямая МК не пересекала отрезок АВ.

      380. В магазине имеются лимоны, апельсины и мандарины, всего 740 кг. Если бы продали 55 кг лимонов, 36 кг апельсинов и 34 кг мандаринов, то оставшиеся массы лимонов, апельсинов и мандаринов оказались бы равными. Сколько килограммов фруктов каждого вида имеется в магазине?

      381. От дома до дачи можно доехать на автобусе, или на электропоезде, или на маршрутном такси. В таблице указано время, которое надо затратить на каждый участок пути. Какое наименьшее время потребуется на дорогу? Каким видом транспорта при этом надо воспользоваться?

      382. Найдите сумму корней уравнений: (x — 18) — 73 = 39


      Задача от мудрой совы

      383. Как с помощью пятилитрового бидона и трёхлитровой банки набрать на берегу реки 4 л воды?


      Этот сайт использует cookie для хранения данных.

      Найдите периметр ABCD

      Элементарная (к 6) Элементарная математика

      Хиди Х.

      спросил 14.06.22

      Найдите периметр многоугольника, образованного этими точками, соединенными;

      A (3,4)

      B (3,1)

      C (8,1)

      D (8,4,4)

      D (8,4,4)

      D (8,4)

      Периметр = ___ единиц

      Подписаться І 3

      Подробнее

      Отчет

      3 ответа от опытных наставников

      Лучший Новейшие Самый старый

      Автор: Лучшие новыеСамые старые

      Сьюзен С. ответил 15.06.22

      Репетитор

      5 (31)

      Я люблю математику и люблю ее преподавать.

      Смотрите таких репетиторов

      Смотрите таких репетиторов

      Привет,

      Координаты даны, 4 точки на координатном графике.

      Шаги:

      1. Отметьте 4 точки на графике.

      (Пожалуйста, убедитесь, что все точки обозначены координатами и буквенными названиями.)

      1. В : Какая фигура у вас есть?

      Ответ: Этот 9Многоугольник 0008 — это прямоугольник . Прямоугольник называется прямоугольником ABCD.

      Здесь важны характеристики:

      Противоположные стороны равны.

      Прямоугольник имеет четыре угла по 90 градусов (прямые углы)

      3. Найдите длину отрезка AB, просто сложив единицы измерения от точки A до точки B. AB=длина AB=3 единицы

      4. Теперь найдите длину отрезка AB длина отрезка ВС=5 ед. Просто считайте от точки B до точки C.

      Примечание: BC= AD=5. И AB=DC=3

      (расстояние от B до C равно расстоянию от A до D, а расстояние от A до B равно расстоянию от D до C).

      В: Что вас просят сделать?

      A: Найдите периметр многоугольника.

      Q : Что такое периметр ?

      A: Периметр — это расстояние вокруг прямоугольника.

      Вы можете сложить все расстояния вокруг или использовать следующую формулу: Р= 2(л) + 2(ж).

      Периметр равен равному удвоенной длине плюс удвоенной ширине .

      Голосовать за 1 голос против

      Подробнее

      Отчет

      Уоллен С. ответил 29.06.22

      Репетитор

      Новое в Византе

      Репетитор средней школы использует математику, чтобы спасти положение.

      Смотрите таких репетиторов

      Смотрите таких репетиторов

      При рассмотрении вопроса нам дали координаты.

      1. Постройте свою декартову плоскость на миллиметровой бумаге.
      2. Нанесите точки на декартову плоскость, и это будет выглядеть как прямоугольник (многоугольник) после того, как каждая точка будет соединена.
      3. Подсчитайте ЕДИНИЦУ между каждой точкой, чтобы узнать, каково расстояние между каждой вершиной.
      4. Расстояние между AB будет равно 5 единицам, а расстояние DC будет равно 3 единицам.
      5. В прямоугольнике противоположные стороны равны, поэтому AB (5 единиц) = CD (5 единиц) и AC (3 единицы) = AD (3 единицы).
      6. Периметр означает расстояние вокруг объекта; Чтобы найти периметр прямоугольника, сложите все длины прямоугольника (AB+BC+CD+DA).

      Периметр прямоугольника будет:

      5+3+5+3 = 16 единиц

      Голосовать за 0 голос против

      Подробнее

      Отчет

      Дерек П. ответил 15.06.22

      Репетитор

      Новое в Византе

      Учитель математики средней школы с 1 годом педагогического стажа

      См. таких репетиторов

      Смотрите таких репетиторов

      Вы можете использовать формулу расстояния, чтобы решить эту проблему. Расстояние = квадратный корень из (x 2 — х 1 ) + (у 2 — у 1 )

      От А до В, (3 — 3) 2 + (1 — 4) 2 2

      2 = + (-3) 2 = 0 + 9 = 9. Квадратный корень из 9 = 3

      От B до C, (8 — 3) 2 + (1 — 1) 2 = 5 2 + 0 2 = 25 + 0 = 25. Квадратный корень из 25 = 5

      От C к D, (8 — 8) 2 + (4 — 1) 2 = 0 2 + 3 2 = 0 + 9 = 9. Квадратный корень из 9= 3

      От D до A, (3 — 8) 2 + (4 — 4) 2 = (-5) 2 + 0 2 = 25 + 0 = 25. Квадратный корень из 25 = 5.

      Теперь сложим все длины сторон вместе: 3 + 5 + 3 + 5 = 16

      Периметр = 16

      Голосовать за 0 голос против

      Подробнее

      Отчет

      Все еще ищете помощи? Получите правильный ответ, быстро.

      Задайте вопрос бесплатно

      Получите бесплатный ответ на быстрый вопрос.
      Ответы на большинство вопросов в течение 4 часов.

      ИЛИ

      Найдите онлайн-репетитора сейчас

      Выберите эксперта и встретьтесь онлайн. Никаких пакетов или подписок, платите только за то время, которое вам нужно.

      В прямоугольнике ABCD длины сторон AB, BC, CD, DA равны \\[\\left( {5x + 2y + 2} \\right)\\] см, \\[\\left( { x + y + 4} \\right)\\] см, \\[\\left( {2x + 5y — 7} \\right)\\] см и \\[\\left( {3x + 2y

      Подсказка: Здесь нужно найти верные утверждения, воспользуемся свойством прямоугольников составить два линейных уравнения с двумя переменными, затем решим их, чтобы найти значения переменных, и, таким образом, получим длины сторон прямоугольника. Затем мы воспользуемся теоремой Пифагора, чтобы найти длину диагоналей. Мы воспользуемся формулами для периметра и площади прямоугольника, чтобы найти их значение. Наконец, мы проверим, какое из данных утверждений верны

      Полный пошаговый ответ:
      Сначала составим два линейных уравнения с двумя переменными, используя предоставленную информацию.
      Мы знаем, что противоположные стороны прямоугольника равны.
      Таким образом, в прямоугольнике ABCD получаем
      \[\begin{array}{l}AB = CD\\BC = DA\end{array}\]
      Подставляя \[AB = \left( {5x + 2y + 2 } \right)\] см и \[CD = \left( {2x + 5y — 7} \right)\] см в уравнении \[AB = CD\], получаем
      \[ \Rightarrow 5x + 2y + 2 = 2х + 5у — 7\]
      Переписывая уравнение, получаем
      \[ \Стрелка вправо 5x + 2y + 2 — 2x — 5y + 7 = 0\]
      Складывая и вычитая одинаковые члены, получаем
      \[ \Стрелка вправо 3x — 3y + 9 = 0\ ]
      Разделив обе части на 3, получим
      \[ \Стрелка вправо x — y + 3 = 0\]…………………..\[\left( 1 \right)\]
      Подставив \[BC = \left( {x + y + 4} \right)\] см и \[DA = \left( {3x + 2y — 11} \right)\] см в уравнении \[BC = DA\], получаем
      \[ \Стрелка вправо x + y + 4 = 3x + 2y — 11\]
      Переписывая уравнение, получаем
      \[ \Стрелка вправо 3x + 2y — 11 — x — y — 4 = 0\]
      Складывая и вычитая подобные члены, получаем
      \[ \Стрелка вправо 2x + y — 15 = 0\]……………… …. \[\left( 2 \right)\]
      Мы можем заметить, что уравнения \[\left( 1 \right)\] и \[\left( 2 \right)\] являются парой линейных уравнений в две переменные.
      Мы будем решать уравнения, чтобы найти значения \[x\] и \[y\].
      Переписывая уравнение \[\left( 1 \right)\], получаем
      \[ \стрелка вправо x = y — 3\]
      Подставляя \[x = y — 3\] в уравнение \[\left( 2 \right )\], получаем
      \[ \Rightarrow 2\left( {y — 3} \right) + y — 15 = 0\]
      Умножив слагаемые по дистрибутивному закону умножения, получим
      \[ \Rightarrow 2y — 6 + y — 15 = 0\]
      Складывая и вычитая одинаковые члены, получаем
      \[ \Стрелка вправо 3y — 21 = 0\]
      Прибавляя 21 с обеих сторон, получаем
      \[ \Стрелка вправо 3y = 21\]
      Разделив обе стороны на 3 получаем
      \[ \Rightarrow y = 7\]
      Подставляя \[y = 7\] в уравнение \[x = y — 3\], получаем
      \[ \Rightarrow x = 7 — 3\ ]
      Вычитая члены в выражении, получаем
      \[ \Стрелка вправо x = 4\]
      Теперь найдем длины сторон прямоугольника.
      Подставив \[x = 4\] и \[y = 7\] в уравнение \[AB = \left( {5x + 2y + 2} \right)\] см, получим
      \[ \Rightarrow AB = 5\left( 4 \right) + 2\left( 7 \right) + 2\]
      Упрощая выражение, получаем
      \[\begin{array}{l} \Rightarrow AB = 20 + 14 + 2\\ \Rightarrow AB = 36{\rm{см}}\end{массив}\]
      Подставляя \[x = 4\] и \[y = 7\] в уравнение \[BC = \left( {x + y + 4}\вправо)\] см, получаем
      \[ \Rightarrow BC = 4 + 7 + 4\]
      Упрощая выражение, получаем
      \[ \Rightarrow BC = 15{\rm{cm}}\]
      Таким образом, получаем
      \[ \Rightarrow AB = CD = 36{\rm{см}}\]
      \[ \Rightarrow BC = DA = 15{\rm{см}}\]
      Мы можем заметить, что одна из пар сторон прямоугольника имеет длину 15 см .
      Следовательно, вариант (а) правильный.
      Теперь найдем длину диагонали AC по теореме Пифагора.
      Мы знаем, что внутренние углы прямоугольника прямые. 92} = 1521\]
      Извлекая квадратный корень из обеих сторон, получаем
      \[ \Стрелка вправо AC = 39\] см
      Диагонали прямоугольника равны.